<<

Leading the world to better

MUSCULOSKELETAL CLINICAL VIGNETTES A CASE BASED TEXT

Department of Orthopaedic Surgery, RCSI Department of General Practice, RCSI Department of , Beaumont Hospital O’Byrne J, Downey R, Feeley R, Kelly M, Tiedt L, O’Byrne J, Murphy M, Stuart E, Kearns G. (2019) Musculoskeletal clinical vignettes: a case based text. Dublin, Ireland: RCSI. ISBN: 978-0-9926911-8-9

Image attribution: istock.com/mashuk

CC Licence by NC-SA

MUSCULOSKELETAL

CLINICAL VIGNETTES

Incorporating history, examination, investigations and management of commonly presenting musculoskeletal conditions

1131

Department of Orthopaedic Surgery, RCSI

Prof. John O'Byrne Department of Orthopaedic Surgery, RCSI Dr. Richie Downey Prof. John O'Byrne Mr. Iain Feeley Dr. Richie Downey Dr. Martin Kelly Mr. Iain Feeley Dr. Lauren Tiedt Dr. Martin Kelly Department of General Practice, RCSI Dr. Lauren Tiedt Dr. Mark Murphy Department of General Practice, RCSI Dr Ellen Stuart Dr. Mark Murphy Department of Rheumatology, Beaumont Hospital Dr Ellen Stuart Dr Grainne Kearns Department of Rheumatology, Beaumont Hospital

Dr Grainne Kearns

2

2

Department of Orthopaedic Surgery, RCSI

Prof. John O'Byrne Department of Orthopaedic Surgery, RCSI Dr. Richie Downey TABLE OF CONTENTS Prof. John O'Byrne Mr. Iain Feeley Introduction ...... 5 Dr. Richie Downey Dr. Martin Kelly General guidelines for musculoskeletal physical Mr. Iain Feeley examination of all ...... 6 Dr. Lauren Tiedt Dr. Martin Kelly Upper limb ...... 10 Department of General Practice, RCSI Example of an upper limb examination ...... 11 Dr. Lauren Tiedt ...... 13 Dr. Mark Murphy Adhesive capsulitis (frozen shoulder) ...... 16 Department of General Practice, RCSI Dr Ellen Stuart Shoulder ...... 19 Dr. Mark Murphy Acromioclavicular joint ...... 22 Department of Rheumatology, Beaumont Hospital Lateral epicondylitis ...... 24 Dr Ellen Stuart Dr Grainne Kearns syndrome ...... 26 Department of Rheumatology, Beaumont Hospital Lower Limb ...... 28 Dr Grainne Kearns Examples of lower limb joint examinations ...... 29 osteoarthritis ...... 34 Hip labral tear ...... 36

Developmental dysplasia of the hip ...... 39

Perthes disease ...... 41 Slipped upper femoral epiphysis ...... 43 Hip fracture ...... 44 Avascular necrosis of the femoral head ...... 47 Transient of the hip ...... 50 of the hip ...... 53 osteoarthritis ...... 55 Anterior cruciate rupture ...... 58 Posterior cruciate ligament rupture ...... 61 Medial collateral ligament injury ...... 64 Meniscal knee injury ...... 67

2 3

2

Deep thrombosis post total ...... 69

Spine ...... 72 Examples of spinal examinations ...... 73 Cervical ...... 75 Whiplash injury ...... 80 Lumbar radiculopathy ...... 82 Spondylolisthesis ...... 85 Lumbar (with possible red flags) ...... 88 Non-specific ...... 92 Spinal stenosis ...... 95 Appendices ...... 98 Appendix 1: Important facts and figures around spinal history and examination ...... 99 Appendix 2: Diagnostic criteria in musculoskeletal ...... 102 Appendix 3: Abbreviations ...... 105 Appendix 4: Notes ...... 107

The symbol * is used in each section when an elaboration on the test/ diagnostic criteria listed is available in the relevant Appendix at the end of the document.

4

Deep vein thrombosis post total hip replacement ...... 69 INTRODUCTION Spine ...... 72 Examples of spinal examinations ...... 73 Cervical radiculopathy ...... 75 This document was generated by the Department of Orthopaedic Surgery and the Department of General Practice Whiplash injury ...... 80 in conjunction with the Department of Rheumatology in the Lumbar radiculopathy ...... 82 Royal College of Surgeons Ireland (RCSI). Spondylolisthesis ...... 85 Lumbar pain (with possible red flags) ...... 88 Musculoskeletal conditions are an extremely common Non-specific low back pain ...... 92 presentation to general practitioners, emergency departments and hospitals. Although the clinical examination for each joint Spinal stenosis ...... 95 involves a similar approach (the ‘Look, Feel, Move’ format), be ...... 98 it the shoulder, hip or knee, many students can find the subtle Appendices differences between each examination difficult. They may also Appendix 1: Important facts and figures around spinal struggle to generate an appropriate , history and examination ...... 99 based upon the history and examination. Appendix 2: Diagnostic criteria in musculoskeletal medicine ...... 102 Commonly presenting musculoskeletal conditions in general practice and orthopaedics are presented in the form of clinical Appendix 3: Abbreviations ...... 105 vignettes in this document. Each vignette starts by Appendix 4: Notes ...... 107 summarising the typical history and examination findings for each condition. The main positive clinical examination findings are outlined in bold and other possible findings in normal font. A list of differential diagnoses are then given followed by sub- headings illustrating why one diagnosis is more likely and The symbol * is used in each section when an elaboration others less so for each case. Appropriate investigations and on the test/ diagnostic criteria listed is available in the management for the general practitioner or orthopaedic relevant Appendix at the end of the document. surgeon are listed to complete each vignette. The Appendices at the end of the document contain some useful additional information.

We hope this learning tool helps develop your musculoskeletal skills during your medical studies in RCSI.

4 5

GENERAL GUIDELINES FOR MUSCULOSKELETAL OF ALL JOINTS

Musculoskeletal disorders are extremely prevalent, causing significant morbidity for patients. For example, up to 20% GP consultations relate to a musculoskeletal condition. Musculoskeletal conditions coexist with many other medical and surgical conditions, necessitating all doctors to have an in depth knowledge of musculoskeletal history taking, examination, diagnosis and management.

The section will briefly outline the general steps of how best to perform a musculoskeletal examination of a joint:

Introduction • Appropriate introduction to patient (full name and role) • Communication: Explain examination and gain consent from patient • Ask the patient if they have any pain before you start • Ask the patient to report any pain or discomfort during the examination • Look at the patient while examining them. Avoid causing them discomfort or pain

Approach • Wash • Expose area under examination adequately. • Ideally the joint above and below should be visible and you can offer to examine them at the end of the examination • Always compare one side with the other for peripheral joint examination. • For simplicity, follow the “Look, Feel, Move, Special Tests” approach to each joint examination.

Look/ Feel/ Move/ Special Tests • When looking, always ‘look’ from the front, the side and the back of any particular joint. • When feeling, ‘feel’ the front, back and side of any joint also. Use the back of your to assess

6

GENERAL GUIDELINES FOR MUSCULOSKELETAL temperature and the pads of your fingers or thumbs to PHYSICAL EXAMINATION OF ALL JOINTS assess tenderness and swelling. Be gentle. • When moving, ‘move’ using Active, Passive and Musculoskeletal disorders are extremely prevalent, causing Resisted movements. Why all three? The significant morbidity for patients. For example, up to 20% GP accumulation of information from each of Active, consultations relate to a musculoskeletal condition. Passive and Resisted movement testing will provide Musculoskeletal conditions coexist with many other medical mounting evidence as to whether the pain is coming and surgical conditions, necessitating all doctors to have an in from arthrogenic (joint, capsule, , bursae), depth knowledge of musculoskeletal history taking, myogenic (muscle, tendon) or other structures (less examination, diagnosis and management. discernible patterns e.g. from spine, visceral structures, functional presentation etc.). The section will briefly outline the general steps of how best to • In reality, ‘move’ assessment to this level of detail is perform a musculoskeletal examination of a joint: only done by physiotherapists or doctors with a special interest in musculoskeletal medicine. The Introduction majority of GPs perform a focused musculoskeletal • Appropriate introduction to patient (full name and role) examination and use a modified Active, Passive and • Communication: Explain examination and gain Resisted movement approach to ‘Move’ depending on consent from patient the joint under assessment. • Ask the patient if they have any pain before you start • Ask the patient to report any pain or discomfort during Active movement the examination • Ask the patient to voluntarily move the joint through a • Look at the patient while examining them. Avoid full (if possible). causing them discomfort or pain • Active movement tests arthrogenic (joint, capsule, ligaments, bursae) and myogenic (muscle, tendon) Approach structures. • Wash hands • Pain, weakness, fear and lack of co-operation may • Expose area under examination adequately. interfere with or limit the patient’s active range of • Ideally the joint above and below should be visible motion. and you can offer to examine them at the end of the examination Passive movement • Always compare one side with the other for peripheral • The examiner puts the joint through a range of motion joint examination. (with no assistance from the patient). • For simplicity, follow the “Look, Feel, Move, Special • Passive movement tests arthrogenic structures (joint, Tests” approach to each joint examination. capsule, ligaments) but theoretically excludes myogenic structures as the patient is relaxed. The Look/ Feel/ Move/ Special Tests patient must be positioned comfortably and the • When looking, always ‘look’ from the front, the side handling of the joint must be good to ensure adequate and the back of any particular joint. relaxation of myogenic structures.

• When feeling, ‘feel’ the front, back and side of any joint also. Use the back of your hand to assess Resisted movement

6 7

• The examiner tests the strength of the myogenic structures around the joint. It is done with the joint in neutral position and the muscle-tendon unit is tested isometrically (no movement of joint required as you want to isolate the neuromuscular unit). • You are looking for pain and/or weakness on testing. • Pain on testing suggests disruption of muscle tendon unit e.g. muscle tear (partial tear, complete tear) or tendinitis. • Weak and painful resisted movement on testing suggests an incomplete muscle tear or tendinitis. • Weak and painless resisted movement on testing suggests either complete rupture of the muscle tendon complex or neurological deficit ( supplying muscle not working). • Resisted movements can be graded e.g. Oxford scale 0-5.

Movement Testing Summary • Problem with Active and Resisted movements – likely myogenic problem • Problem with Active and Passive movements – likely arthrogenic problem • Problem with Active, Passive and Resisted movements – consider rheumatological disease, systemic disease, psychosocial issues, neurological compromise (see red and yellow flags)

Special tests • Special tests are specific tests for structural damage in each anatomical joint • For example, with a knee joint examination, the following are ‘special tests’: o Swelling: test o Ligaments: ACL, Lachmanns, PCL, MCL, LCL. Pivot shift test o Menisici: McMurrays o Patella: Patellar Apprehension

For the purposes of exams

8

• The examiner tests the strength of the myogenic • Offer to examine joint above and below (remember for structures around the joint. It is done with the joint in example that right Hip OA can cause pain down right neutral position and the muscle-tendon unit is tested anterior which can go down as far as the knee isometrically (no movement of joint required as you joint). want to isolate the neuromuscular unit). • Offer to check neurovascular status of patients’ limbs • You are looking for pain and/or weakness on testing. (e.g. distal sensation, peripheral ) in addition to • Pain on testing suggests disruption of muscle tendon joint examination. unit e.g. muscle tear (partial tear, complete tear) or • When doing spinal examinations, add a neurological tendinitis. examination of the upper limbs to the cervical spine • Weak and painful resisted movement on testing examination, and add a of suggests an incomplete muscle tear or tendinitis. the lower limbs to the lumbar spine examination • Weak and painless resisted movement on testing suggests either complete rupture of the muscle tendon complex or neurological deficit (nerve supplying muscle not working). • Resisted movements can be graded e.g. Oxford scale 0-5.

Movement Testing Summary • Problem with Active and Resisted movements – likely myogenic problem • Problem with Active and Passive movements – likely arthrogenic problem • Problem with Active, Passive and Resisted movements – consider rheumatological disease, systemic disease, psychosocial issues, neurological compromise (see red and yellow flags)

Special tests • Special tests are specific tests for structural damage in each anatomical joint • For example, with a knee joint examination, the following are ‘special tests’: o Swelling: Patellar tap test o Ligaments: ACL, Lachmanns, PCL, MCL, LCL. Pivot shift test o Menisici: McMurrays o Patella: Patellar Apprehension

For the purposes of exams

8 9

UPPER LIMB

10

EXAMPLE OF AN UPPER LIMB JOINT UPPER LIMB EXAMINATION

Shoulder joint examination summary

Look: • Skin – scars, erythema • Soft Tissue – swelling, muscle wasting e.g. deltoid/scapular muscles • – deformity e.g. clavicular deformity or winging of scapulae (enhance by getting patient to do standing press-up against wall)

Feel: • Skin – temperature • Soft Tissue – swelling, tenderness e.g. Acromioclavicular joint tenderness (possible OA), tenderness around greater tuberosity humerus (possible impingement) • Bone – tenderness, deformity

Move: Active and Passive movement (range of motion) • Flexion 180° • Extension 50° (opposite direction to Flexion) • Abduction 180° • Lateral Rotation 60° • Medial Rotation – thumb to inferior border scapula

o Check pain throughout e.g. arc of pain with abduction (classic sign impingement) or e.g. unable to hold the in abduction (weakness due to possible supraspinatus tear)

Resisted movement • Check rotator cuff muscle strength/ reproduction of pain on resisted movements of shoulder (flexion,

10 11

extension, abduction, adduction, medial and lateral rotation). o e.g. Resisted lateral rotation – by side, in mid-position, resist movement (at the ) of forearm laterally.

Special tests: a) Rotator cuff impingement: if you have palpated tenderness around the greater tuberosity and found an arc of pain with active abduction and pain reproduced on resisted abduction, you may decide to test more specifically for impingement. • e.g. Jobe’s test – Empty Can test – position arm so that it is abducted to 90°, horizontally adducted 30° and medially rotate arm so it looks like you are emptying an imaginary can, resist further abduction of the upper limb (resist at the wrist joint).

b) If patient has swelling over AC joint area or is pointing specifically to pain over this area, consider doing the ‘scarf test’ – cross arm adduction of shoulder – place the patient’s hand over their opposite shoulder as if you are positioning a scarf over that shoulder.

c) If patient complains of pain with overhead activity or has a history of shoulder subluxation/ dislocation, do the instability tests e.g. sulcus test – distract humerus out of glenoid cavity gently by applying downward traction at the elbow joint.

d) Consider the apprehension test in someone with a history of trauma or subluxation/dislocation – with arm held at 90 Abduction, slowly laterally rotate arm while checking patients face for signs of apprehension.

12

extension, abduction, adduction, medial and lateral SHOULDER OSTEOARTHRITIS rotation). o e.g. Resisted lateral rotation – elbow by side, History: forearm in mid-position, resist movement (at A 67-year-old retired electrician presents to his GP the wrist) of forearm laterally. complaining of significant right shoulder pain. The pain began gradually, has become worse over the last 2 years and has Special tests: forced him to give up playing tennis. The pain is worse with a) Rotator cuff impingement: if you have palpated overhead activity and can disturb his sleep after a game of tenderness around the greater tuberosity and found tennis. Of note, he sustained a proximal right humeral fracture an arc of pain with active abduction and pain following a fall from a horse 20 years ago, which was reproduced on resisted abduction, you may decide to managed conservatively. He has hypertension and test more specifically for impingement. hypercholesterolaemia. • e.g. Jobe’s test – Empty Can test – position arm so that it is abducted to 90°, What would you look for on examination (clinical findings from this case in bold): horizontally adducted 30° and medially rotate arm so it looks like you are emptying an imaginary can, resist further Look/ inspection Reduced muscle bulk right deltoid abduction of the upper limb (resist at the (disuse ) wrist joint). Feel Tenderness over right gleno- humeral joint b) If patient has swelling over AC joint area or is pointing Crepitus specifically to pain over this area, consider doing the Move Active Reduced and painful active range ‘scarf test’ – cross arm adduction of shoulder – place of motion of R shoulder (in the patient’s hand over their opposite shoulder as if particular lateral rotation R you are positioning a scarf over that shoulder. shoulder) Passive Reduced and painful passive range c) If patient complains of pain with overhead activity or of motion of right shoulder (in has a history of shoulder subluxation/ dislocation, do particular lateral rotation right the instability tests e.g. sulcus test – distract shoulder) humerus out of glenoid cavity gently by applying Resisted No pain or weakness reproduced on downward traction at the elbow joint. resisted movement Right shoulder Special tests for this Scarf test for AC Joint*: Normal d) Consider the apprehension test in someone with a joint examination Empty can test*: Normal history of trauma or subluxation/dislocation – with arm Apprehension test for instability in held at 90 Abduction, slowly laterally rotate arm while ligaments*: Not performed as history checking patients face for signs of apprehension. not suggestive of subluxation Other Possible osteoarthritis changes hands Cervical spine examination: Normal Neurological examination upper limb: Normal

12 13

What are your differential diagnoses? • Osteoarthritis right glenohumeral (shoulder) joint • Another capsular (“joint”) glenohumeral condition such as frozen shoulder (“adhesive capsulitis”), or gout. • Rotator cuff pathology

Why is osteoarthritis (OA) R shoulder likely? • History: Pain, gradual onset, worse with activity, history of previous fracture R arm (? intra-articular) • Examination: Reduced and painful active and passive range of motion (in particular lateral rotation limited)

Why are frozen shoulder/ rheumatoid arthritis (RA)/ gout unlikely? • History: Frozen shoulder tends to have a painful and then a stiff phase over an 18-month period, this history does not tie in with this. There is no previous history of RA, no family history of RA, asymmetrical joint presentation (normally symmetrical joints affected with RA). Gout is high up in the differential given his metabolic risk factors but is more common in feet/ ankles/ hands/ than • Examination: Frozen shoulder may have similar clinical examination (minus possible OA changes elsewhere on body). There are no other clinical findings suggestive of RA or gout e.g. RA changes in hands, gouty tophi etc.

Why is rotator cuff pathology unlikely? • No pain or weakness reproduced with resisted movements

What investigations would you do? • In the GP surgery, order bloods (routine plus arthritis screen plus ESR/ CRP/ uric acid) and X-ray right shoulder • X-ray: AP, true AP and axillary X-ray right shoulder. X-ray may show subchondral sclerosis and/or

14

osteophytes at inferior aspect of humeral head What are your differential diagnoses? ("goat's beard”). Rheumatoid Arthritis may show a • Osteoarthritis right glenohumeral (shoulder) joint more erosive pattern on X-ray • Another capsular (“joint”) glenohumeral condition such • CT right shoulder: Indicated in inflammatory arthritis if as frozen shoulder (“adhesive capsulitis”), rheumatoid large bony defects are present on X-ray arthritis or gout. • MRI right shoulder: May be indicated to evaluate • Rotator cuff pathology rotator cuff tendon (not needed in this case)

Why is osteoarthritis (OA) R shoulder likely? What are the management options? • History: Pain, gradual onset, worse with activity, a) Initial management by the GP includes conservative history of previous fracture R arm (? intra-articular) treatments: • Examination: Reduced and painful active and passive • Analgesia as per WHO Pain ladder (start with range of motion (in particular lateral rotation limited) Paracetamol and NSAIDs) • Refer for physiotherapy Why are frozen shoulder/ rheumatoid arthritis (RA)/ gout • Consider intra-articular injection of steroid and local unlikely? anaesthetic if severe • History: Frozen shoulder tends to have a painful and • Orthopaedic referral (if acceptable) then a stiff phase over an 18-month period, this b) Orthopaedic secondary care management: history does not tie in with this. There is no previous • Conservative: NSAIDs, physiotherapy, intra-articular history of RA, no family history of RA, asymmetrical steroid injections joint presentation (normally symmetrical joints • Operative: , affected with RA). Gout is high up in the differential given his metabolic risk factors but is more common in feet/ ankles/ hands/ wrists than shoulders • Examination: Frozen shoulder may have similar clinical examination (minus possible OA changes elsewhere on body). There are no other clinical findings suggestive of RA or gout e.g. RA changes in hands, gouty tophi etc.

Why is rotator cuff pathology unlikely? • No pain or weakness reproduced with resisted movements

What investigations would you do? • In the GP surgery, order bloods (routine plus arthritis screen plus ESR/ CRP/ uric acid) and X-ray right shoulder • X-ray: AP, true AP and axillary X-ray right shoulder. X-ray may show subchondral sclerosis and/or

14 15

ADHESIVE CAPSULITIS (FROZEN SHOULDER)

History: A 57-year-old lady presents to her GP complaining of a 3- month history of right shoulder pain. The pain has not gotten any worse over time but has not settled despite painkillers. It is intermittent and aching in quality. She now has difficulties with washing and dressing in the morning. Her sleep is disturbed by pain when she lies on the right arm during the night.

She has type 1 mellitus, taking insulin, with well- controlled blood glucose levels. She has no other past of note.

What would you look for on examination (clinical findings from this case in bold):

Look/ inspection Reduced muscle bulk right deltoid (disuse atrophy) Feel No tenderness over right gleno- humeral joint Move Active Reduced and painful active range of motion of right shoulder (in particular lateral rotation right shoulder followed by abduction followed by medial rotation) Passive Reduced and painful passive range of motion of right shoulder (in particular lateral rotation right shoulder followed by abduction followed by medial rotation) Resistant and painful ‘end feel’ but no crepitus Resisted No pain reproduced on resisted movement right shoulder Special tests for this Nil joint examination Other Nil

16

ADHESIVE CAPSULITIS (FROZEN SHOULDER) What are your differential diagnoses? • Adhesive capsulitis History: • Rotator cuff pathology A 57-year-old lady presents to her GP complaining of a 3- • Gleno-humeral arthritis month history of right shoulder pain. The pain has not gotten any worse over time but has not settled despite painkillers. It Why is adhesive capsulitis likely? is intermittent and aching in quality. She now has difficulties • History: Painful and stiff right shoulder, history of with washing and dressing in the morning. Her sleep is diabetes (Type 1 in this case). disturbed by pain when she lies on the right arm during the • Examination: Reduced and painful active and passive night. range of motion R shoulder, no tenderness

She has type 1 diabetes mellitus, taking insulin, with well- Why are other differential diagnoses unlikely? controlled blood glucose levels. She has no other past • Rotator cuff pathology unlikely: As there is no painful medical history of note. arc during active range of motion, there is limited passive range of motion (would probably be full if What would you look for on examination (clinical findings rotator cuff pathology present) and there is no pain from this case in bold): reproduced on resisted movement testing. There is no tenderness around R shoulder either. Look/ inspection Reduced muscle bulk right deltoid • Gleno-humeral osteoarthritis unlikely: Difficult to (disuse atrophy) distinguish between these two presentations and they Feel No tenderness over right gleno- may co-present. Gleno-humeral osteoarthritis tends to humeral joint be slower in onset than adhesive capsulitis and not as Move Active Reduced and painful active range painful early on in the disease trajectory. It has a of motion of right shoulder (in similar clinical examination except tenderness may be particular lateral rotation right present also (different to adhesive capsulitis) shoulder followed by abduction followed by medial rotation) What investigations would you do? Passive Reduced and painful passive range In the GP surgery: of motion of right shoulder (in • Refer X-ray right shoulder to check for concomitant particular lateral rotation right osteoarthritis or shoulder followed by abduction • Consider sending bloods for FBC, ESR, CRP, followed by medial rotation) rheumatoid screen to out-rule differential diagnosis if Resistant and painful ‘end feel’ but suspected no crepitus • If diagnostic doubt, MRI shoulder may help to out-rule Resisted No pain reproduced on resisted rotator cuff pathology movement right shoulder If referred to Orthopaedic OPD Special tests for this Nil • May be referred for MR : Loss of axillary joint examination recess indicates of joint capsule Other Nil What are the management options?

16 17

a) Initial management by the GP includes conservative treatments: • Analgesia as per WHO Pain ladder (start with Paracetamol and NSAIDs) • Refer for physiotherapy • Consider intra-articular injection of steroid and local anaesthetic if severe • Orthopaedic referral b) Orthopaedic secondary care management: • Non-operative: Analgesia, physiotherapy, intra- articular steroid injections • Operative: Manipulation under anaesthesia (MUA), Arthroscopic release

18 a) Initial management by the GP includes conservative SHOULDER ROTATOR CUFF PATHOLOGY treatments: • Analgesia as per WHO Pain ladder (start with History: Paracetamol and NSAIDs) A 24-year-old PhD student presents to her GP with right • Refer for physiotherapy shoulder pain brought on after playing in a badminton • Consider intra-articular injection of steroid and local tournament at the weekend. Pain is over anterior aspect R anaesthetic if severe shoulder. Aggravated by overhead activity. Eased with rest. • Orthopaedic referral Disturbs sleep if lies on right side. Otherwise well. No history b) Orthopaedic secondary care management: of shoulder problems previously. No neurological symptoms. • Non-operative: Analgesia, physiotherapy, intra- articular steroid injections What would you look for on examination (clinical findings • Operative: Manipulation under anaesthesia (MUA), from this case in bold): Arthroscopic release Look/ inspection Nil of note on inspection Feel Tenderness over anterior aspect right glenohumeral joint (site of insertion of rotator cuff tendons) Move Active Painful full range of active range of motion of right shoulder (in particular arc of pain on abduction shoulder) Passive Less pain on passive range of motion of right shoulder Normal end feel and no crepitus Resisted Painful resisted movements around glenohumeral joint (but normal power). Pain especially on resisted movements of Abduction (when the right arm is by her side) and lateral rotation. ABduction- assesses supraspinatus Lateral rotation- assesses infraspinatus/ teres minor Medial rotation- assesses subscapularis Special tests for this Scarf test for AC Joint*: Normal joint examination Empty can test*: Painful Apprehension test for instability in ligaments*: Not performed as history

18 19

not suggestive of subluxation Other Nil

What are your differential diagnoses? • • Adhesive capsulitis • Glenohumeral arthritis • Cervical radiculopathy

Why is rotator cuff tear likely? • History: Sudden onset pain, related to overhead activity, no history of trauma otherwise • Examination: Arc of pain on active range of motion, pain/ weakness on resisted testing

Why is adhesive capsulitis unlikely? • No restriction to active and passive range of motion of right shoulder

Why is glenohumeral arthritis unlikely? • No restriction to active and passive range of motion of right shoulder

Why is cervical radiculopathy unlikely? • Would expect pain going from neck down arm to below elbow. No other clinical findings of note e.g. no sensory disturbance, no motor weakness, no loss etc.

What investigations would you do? • GP: No investigations may be needed if confident of diagnosis. If diagnostic doubt is persisting, consider: • X-ray (infrequently done in these cases): Looking for calcific tendonitis, calcification in the coracohumeral ligament, proximal migration of humerus seen with chronic rotator cuff tear • MRI (may be used if diagnostic uncertainty or diagnostic urgency): Diagnostic of rotator cuff pathology and evaluates muscle quality, size, shape, and degree of retraction of tear

20

not suggestive of subluxation • (most efficient and cost effective in correct Other Nil hands): Diagnostic of rotator cuff tear

What are your differential diagnoses? What are the management options? • Rotator cuff tear a) Initial management by the GP includes conservative • Adhesive capsulitis treatments: • Glenohumeral arthritis • Analgesia as per WHO Pain ladder (start with • Cervical radiculopathy Paracetamol and NSAIDs) • Refer for physiotherapy (usually sufficient to manage Why is rotator cuff tear likely? condition) • History: Sudden onset pain, related to overhead • Orthopaedic referral if severe or persistent pain/ activity, no history of trauma otherwise restriction or severe pathology of rotator cuff on MRI • Examination: Arc of pain on active range of motion, Shoulder pain/ weakness on resisted testing b) Orthopaedic secondary care management: • Non-operative: Analgesia; Physiotherapy Why is adhesive capsulitis unlikely? • Operative: Arrange for Arthroscopic subacromial • No restriction to active and passive range of motion of decompression, rotator cuff repair, arthroscopic or right shoulder open tendon transfers

Why is glenohumeral arthritis unlikely? • No restriction to active and passive range of motion of right shoulder

Why is cervical radiculopathy unlikely? • Would expect pain going from neck down arm to below elbow. No other clinical findings of note e.g. no sensory disturbance, no motor weakness, no reflex loss etc.

What investigations would you do? • GP: No investigations may be needed if confident of diagnosis. If diagnostic doubt is persisting, consider: • X-ray (infrequently done in these cases): Looking for calcific tendonitis, calcification in the coracohumeral ligament, proximal migration of humerus seen with chronic rotator cuff tear • MRI (may be used if diagnostic uncertainty or diagnostic urgency): Diagnostic of rotator cuff pathology and evaluates muscle quality, size, shape, and degree of retraction of tear

20 21

ACROMIOCLAVICULAR JOINT INJURY

History: A 16-year-old right-handed boy presents to the Emergency Department with his father complaining of severe pain over his right acromioclavicular joint following a tackle during a rugby match in school that morning, during which he fell onto his outstretched right hand. He has pain with movement of the right arm but denies other symptoms. He has no background medical history of note.

What would you look for on examination (clinical findings from this case in bold):

Look/ inspection Deformity (lump palpated/ visualised over the acromioclavicular joint) Tenting of the skin (indicates impending open fracture) Feel Tenderness over acromioclavicular joint Move Active Reduced ROM right shoulder due to pain Passive Reduced ROM right shoulder due to pain Resisted Pain with resisted movements right shoulder Special tests for this Scarf test* – positive joint examination Other Upper limb neurological examination: normal Vascular assessment of upper limb: normal

What are your differential diagnoses? • Acromioclavicular joint sprain • Acromioclavicular joint subluxation • Acromioclavicular joint dislocation

Why is acromioclavicular joint sprain likely?

22

ACROMIOCLAVICULAR JOINT INJURY • Typical history and examination findings e.g. lump/ tenderness over the clavicle, reduced range of motion History: of right shoulder as a result A 16-year-old right-handed boy presents to the Emergency Department with his father complaining of severe pain over Why other diagnoses are unlikely? his right acromioclavicular joint following a tackle during a • Acromioclavicular joint subluxation/dislocation: need rugby match in school that morning, during which he fell onto to out-rule with imaging his outstretched right hand. He has pain with movement of the right arm but denies other symptoms. He has no background What investigations would you perform? medical history of note. • X-ray of clavicle/ Acromioclavicular joint

What would you look for on examination (clinical findings What are the management options in the Emergency from this case in bold): Department? Orthopaedic referral: Orthopaedic team will decide whether Look/ inspection Deformity (lump palpated/ non-operative or operative management appropriate. visualised over the • Non-operative: Sling, with introduction of ROM acromioclavicular joint) at 2-4weeks. Analgesia as per WHO pain Tenting of the skin (indicates ladder (start with paracetamol and NSAIDs). impending open fracture) Physiotherapy Feel Tenderness over acromioclavicular • Operative: Operative intervention may be warranted if joint the coracoclavicular ligaments are disrupted Move Active Reduced ROM right shoulder due to pain Passive Reduced ROM right shoulder due to pain Resisted Pain with resisted movements right shoulder Special tests for this Scarf test* – positive joint examination Other Upper limb neurological examination: normal Vascular assessment of upper limb: normal

What are your differential diagnoses? • Acromioclavicular joint sprain • Acromioclavicular joint subluxation • Acromioclavicular joint dislocation

Why is acromioclavicular joint sprain likely?

22 23

LATERAL EPICONDYLITIS

History: A 40 year old carpenter who is also a motor bike fanatic presents to your GP surgery complaining of pain on the outside of his elbow. The pain is at its worst after a day’s work or when he comes off his bike after a long journey. The pain is relieved by rest. On further questioning, the pain radiates down the forearm. When asked to pinpoint the pain, he puts his finger just distal to the lateral epicondyle. He has no other medical conditions.

What would you look for on examination (clinical findings from this case in bold):

Look/ inspection Nil obvious on inspection Feel Pain on palpitation 1cm distal to the lateral epicondyle Move Active Pain on wrist extension Passive Minimal pain on passive movement Resisted Pain exacerbated with resisted wrist extension Special tests for this No special tests applicable joint examination Other Neurological examination of hand, and an examination of the C spine to rule out cervical radiculopathy or impingement

What are your differential diagnoses? • / Lateral epicondylitis • Cervical radiculopathy • Radial nerve impingement

Why is Tennis Elbow likely? • Typical history of pain and a career and hobby which involve repetitive wrist extension. The point tenderness distal to the lateral epicondyle and pain on resisted wrist extension is quite sensitive for the condition

24

LATERAL EPICONDYLITIS Why other diagnoses are unlikely? History: • Cervical disc disease unlikely as: No pain in cervical A 40 year old carpenter who is also a motor bike fanatic spine. Normal cervical spine examination. Normal presents to your GP surgery complaining of pain on the neurological examination upper limb outside of his elbow. The pain is at its worst after a day’s work • Radial nerve impingement unlikely as: Normal distal or when he comes off his bike after a long journey. The pain is neurological exam with full power of wrist extension relieved by rest. On further questioning, the pain radiates down the forearm. When asked to pinpoint the pain, he puts What investigations would you perform? his finger just distal to the lateral epicondyle. He has no other Tennis elbow is a clinical diagnosis. No investigations may be medical conditions. needed • Consider X-ray if osteochondral defect or OA is in What would you look for on examination (clinical findings differential (not in this case) from this case in bold): • Nerve conduction studies can be helpful if concern for radial nerve entrapment Look/ inspection Nil obvious on inspection • Ultrasound may be useful for the diagnosis of a partial Feel Pain on palpitation 1cm distal to tendon tear the lateral epicondyle Move Active Pain on wrist extension What are the management options? Passive Minimal pain on passive movement a) Initial management by the GP includes: Resisted Pain exacerbated with resisted • Analgesia as per WHO pain ladder (start with wrist extension Paracetamol and NSAIDs) Special tests for this No special tests applicable • Watchful waiting – tends to resolve spontaneously joint examination within 52 weeks Other Neurological examination of hand, • Refer for physiotherapy – may include check for and an examination of the C spine overuse, use of counter force bracing to rule out cervical radiculopathy or • Referral to Orthopaedics if problem persists radial nerve impingement b) Orthopaedic team in secondary care: • Non-operative: Rest, , Physiotherapy, What are your differential diagnoses? steroid injection into tendon • Tennis elbow/ Lateral epicondylitis • Operative: Reserved for refractory cases (6 months of • Cervical radiculopathy conservative care). Involves arthroscopic debridement • Radial nerve impingement of pathological tissue within tendon of extensor carpi radialis brevis (ECRB). This can also be done as an Why is Tennis Elbow likely? open procedure • Typical history of pain and a career and hobby which involve repetitive wrist extension. The point tenderness distal to the lateral epicondyle and pain on resisted wrist extension is quite sensitive for the condition

24 25

CARPAL TUNNEL SYNDROME

History: A 34-year-old musician presents to the orthopaedic OPD complaining of a 6-month history of pain in her left hand. The pain keeps her awake at night and is not relieved by simple analgesia. Occasionally she suffers pins and needles in the same hand. She has a raised BMI and is on Eltroxin. She says she is fully compliant with her Eltroxin therapy. Her Thyroid Function Tests were checked last month and were normal.

What would you look for on examination (clinical findings from this case in bold):

Look/ inspection Wasting of thenar muscles Feel Paraesthesia/ Numbness in distribution in hand Move Active Nil of note Passive Nil of note Resisted Nil of note Special tests for this Phalen’s test: wrist volar flexion for joint examination ~60 sec produces symptoms Tinel's test: provocative tests performed by tapping the median nerve over the volar carpal tunnel Other Neurological examination of hand: possible reduced sensation over palmar radial surface thumb and index finger with reduction in power of thumb (Abduction) Cervical spine examination: Normal

What are your differential diagnoses? • Carpal tunnel syndrome • Cervical disc disease • • Multiple Sclerosis

Why is carpal tunnel syndrome likely?

26

CARPAL TUNNEL SYNDROME • Typical history of pain (nocturnal) in median nerve distribution in hand and positive Tinel’s sign History: A 34-year-old musician presents to the orthopaedic OPD Why other diagnoses are unlikely? complaining of a 6-month history of pain in her left hand. The • Cervical disc disease unlikely as: No pain in cervical pain keeps her awake at night and is not relieved by simple spine. Normal cervical spine examination. Normal analgesia. Occasionally she suffers pins and needles in the neurological examination upper limb same hand. She has a raised BMI and is on Eltroxin. She • Hypothyroidism unlikely as: Normal TFTs last month. says she is fully compliant with her Eltroxin therapy. Her Fully compliant with Eltroxin Thyroid Function Tests were checked last month and were • MS unlikely as: No evidence of symptomatic episodes normal. that are separated in space and time

What would you look for on examination (clinical findings What investigations would you perform? from this case in bold): • Carpal tunnel syndrome is a clinical diagnosis. No investigations may be needed Look/ inspection Wasting of thenar muscles • Consider X-ray hand if OA is in differential (not in this Feel Paraesthesia/ Numbness in median case) nerve distribution in hand • Nerve conduction studies can be helpful if uncertain Move Active Nil of note clinical diagnosis Passive Nil of note • Bloods can be considered (fasting glucose Resisted Nil of note Rheumatoid factor) if looking for a cause Special tests for this Phalen’s test: wrist volar flexion for • test needs to be considered in woman of joint examination ~60 sec produces symptoms childbearing age Tinel's test: provocative tests performed by tapping the median What are the management options? nerve over the volar carpal tunnel a) Initial management by the GP includes: Other Neurological examination of hand: • Analgesia as per WHO pain ladder (start with possible reduced sensation over Paracetamol and NSAIDs) palmar radial surface thumb and • Refer for physiotherapy – may include night splints, index finger with reduction in power check for overuse e.g. musician/ hours of practice/ of thumb (Abduction) performing etc. Cervical spine examination: Normal • Referral to Orthopaedics if problem persists b) Orthopaedic team in secondary care: What are your differential diagnoses? • Non-operative: Rest, Analgesics, Physiotherapy, • Carpal tunnel syndrome steroid injection • Cervical disc disease • Operative: Carpal tunnel release (open versus • Hypothyroidism endoscopic) • Multiple Sclerosis

Why is carpal tunnel syndrome likely?

26 27

LOWER LIMB

28

EXAMPLES OF LOWER LIMB JOINT LOWER LIMB EXAMINATIONS

Hip joint examination summary

Look: • Skin: Erythema, scars (e.g. post Total Hip Replacement) • Soft Tissue: Swelling, muscle wasting e.g. Gluteal muscle wasting • Bone: Alignment, lumbar lordosis, fixed flexion deformity of hip • Gait: , antalgic gait, use of walking aids?

Feel: • Skin: temperature, distal sensation • Soft Tissue: swelling, tenderness, peripheral pulses • Bone: greater trochanter tenderness (can be a sign of trochanteric )

Move: Active and Passive movement (range of motion) • Modified – Flexion 130°, Extension 0° o In supine position, flex the unaffected hip. If the affected hip rises from the plinth, this indicates a loss of extension in the hip i.e. presence of a fixed flexion deformity (a ‘positive’ Thomas test). • Rotation, in supine postion, hip flexed at 90°, knee flexed at 90° (Medial Rotation 45°, Lateral Rotation 45°) • Abduction 45°, Adduction 30°

Resisted movement • Check hip muscle strength/ any pain on resisted movements of the hip joint e.g. Flexion, Extension, Abduction, Adduction, Medial and Lateral Rotation

28 29

Special tests:

• Trendelenburg Test o Standing on normal leg only, gluteus medius on supported leg contracts and elevates the on the unsupported side. Elevation of the unsupported leg is normal, indicating a strong functional gluteus medius on the supported side (the standing leg). This is a negative Trendelenburg sign. o However if the unsupported side drops (pelvis drops on the non-standing leg), this indicates that the gluteus medius on the standing leg may non-functioning or weak. This is called a positive Trendelenburg sign.

Paediatric

Barlow test • Dislocates a dislocatable hip by adduction and posterior pressure on a flexed

Ortolani Test • Reduces a dislocated hip by abduction and elevation of flexed femur

30

Knee joint examination summary

Special tests: Look: • Skin: Scars, erythema • Trendelenburg Test • Soft Tissue: Swelling, joint effusion, muscle wasting o Standing on normal leg only, gluteus medius e.g. quadriceps wasting with OA knee on supported leg contracts and elevates the • : Valgus or varus deformities pelvis on the unsupported side. Elevation of • Gait: Antalgic gait the unsupported leg is normal, indicating a strong functional gluteus medius on the Feel: supported side (the standing leg). This is a • Skin: Temperature negative Trendelenburg sign. • Soft Tissue: Swelling, effusion – Stroke test, o However if the unsupported side drops (pelvis tenderness e.g. patellar tendon tenderness (can be a drops on the non-standing leg), this indicates sign with patellar bursitis or ), that the gluteus medius on the standing leg collateral ligament tenderness (e.g. can be a sign with may non-functioning or weak. This is called a collateral ligament sprain), joint line tenderness positive Trendelenburg sign. • Bone: Bone tenderness

Move: Paediatric hip examination Active and Passive movement (range of motion)

• Flexion (130 ) and Extension (0 ) Barlow test ° °

• Dislocates a dislocatable hip by adduction and Resisted movement posterior pressure on a flexed femur • Check knee muscle strength/ any pain on resisted

movements of the knee joint e.g. Flexion, Extension

• Reduces a dislocated hip by abduction and elevation Special tests: of flexed femur

a) Anterior and Posterior Draw Test for anterior and posterior cruciate ligament test

• Anterior : patient supine, knee flexed 90˚ (check hamstrings relaxed), examiner applies posteroanterior force (anterior glide) on the . Excessive laxity compared to the opposite side indicates a positive test. • Posterior Drawer Test: same position as above, examiner applies an anteroposterior force to the tibia. Tests integrity of the PCL. Excessive laxity compared to opposite side indicates a positive test.

30 31

b) Medial and Lateral Collateral Ligament test • Patient supine, knee in 30˚ flexion, examiner applies alternate valgus (abduction) force and varus (adduction) force to the knee joint. Valgus force stresses the Medial Collateral ligament, varus force stresses the Lateral Collateral Ligament. Excessive laxity and/or pain compared to opposite side indicates a positive test.

c) McMurray’s Test for medial and lateral • Patient supine, examiner passively fully flexes knee in lateral rotation while palpating the medial joint line. The joint is then moved from full flexion to extension with rotation maintained. A positive test is when a click is felt over the medial joint line, indicating a medial meniscal tear. The same test can be done in flexion/medial rotation for the lateral meniscus.

d) Patellar Apprehension Test • The patient should be lying supine with their knee at 30˚ flexion and the quadriceps should be relaxed. On examination one should carefully glide the patella laterally closely observing apprehension in the patient. A positive test is the presence of this reaction from the patient.

e) • This is another test used to check the Anterior Cruciate Ligament (along with the anterior drawer test). The Lachman test is usually performed with the knee held in 15 degrees of flexion and in lateral rotation (which relaxes the iliotibial band). When the examiner chooses to examine the right knee, they should hold the inner aspect of the upper part of the right lower leg with their right hand and the outer aspect of the lower part of the thigh with their left hand. On anteriorly gliding the tibia, the examiner will feel a firm end-feel if the Anterior Cruciate ligament is

32

intact. If the end-feel is loose and not firm, this is a b) Medial and Lateral Collateral Ligament test positive test. Always compare both . • Patient supine, knee in 30˚ flexion, examiner applies alternate valgus (abduction) force and varus (adduction) force to the knee joint. Valgus force stresses the Medial Collateral ligament, varus force stresses the Lateral Collateral Ligament. Excessive laxity and/or pain compared to opposite side indicates a positive test. c) McMurray’s Test for medial and lateral meniscus • Patient supine, examiner passively fully flexes knee in lateral rotation while palpating the medial joint line. The joint is then moved from full flexion to extension with rotation maintained. A positive test is when a click is felt over the medial joint line, indicating a medial meniscal tear. The same test can be done in flexion/medial rotation for the lateral meniscus. d) Patellar Apprehension Test • The patient should be lying supine with their knee at 30˚ flexion and the quadriceps should be relaxed. On examination one should carefully glide the patella laterally closely observing apprehension in the patient. A positive test is the presence of this reaction from the patient. e) Lachman Test • This is another test used to check the Anterior Cruciate Ligament (along with the anterior drawer test). The Lachman test is usually performed with the knee held in 15 degrees of flexion and in lateral rotation (which relaxes the iliotibial band). When the examiner chooses to examine the right knee, they should hold the inner aspect of the upper part of the right lower leg with their right hand and the outer aspect of the lower part of the thigh with their left hand. On anteriorly gliding the tibia, the examiner will feel a firm end-feel if the Anterior Cruciate ligament is

32 33

HIP OSTEOARTHRITIS

History: A 69-year-old female presents to you in General Practice complaining of left sided and of 3 years duration. She informs you that the pain is getting progressively worse and now keeps her awake at night. She finds it hard to put on her socks and shoes and clip her toenails. She reports no history of trauma.

What would you look for on examination (clinical findings from this case in bold):

Look/ inspection Trendelenburg gait, Feel No tenderness. Measure leg lengths for discrepancy (True and apparent)* Move Active Decreased range of motion of left hip (all movements) Passive Decreased range of motion of left hip (in particular medial rotation of hip – stiffness at end of available range) Resisted Nil of note Special tests for this Trendelenburg test* joint examination Thomas’s test* Other BMI (29); Normal

What are your differential diagnoses? • Hip OA • Knee OA • Nerve root Impingement • Trochanteric bursitis

Why is Hip OA likely? • History: Site of pain left groin and left knee • Examination findings: Trendelenburg gait; reduced AROM and PROM hip. Positive Thomas’s test left hip

Why are the other differential diagnoses unlikely?

34

HIP OSTEOARTHRITIS • Knee OA unlikely as: o Knee examination is normal History: o Pain is radiating from groin to knee A 69-year-old female presents to you in General Practice • Nerve Root Impingement unlikely as: complaining of left sided groin and knee pain of 3 years o Associated hip stiffness duration. She informs you that the pain is getting o Pattern of pain referral (nerve root progressively worse and now keeps her awake at night. She impingement tends to cause buttock and finds it hard to put on her socks and shoes and clip her posterior thigh pain) toenails. She reports no history of trauma. • Trochanteric bursitis o Associated with advanced hip OA, but can What would you look for on examination (clinical findings occur in isolation. Tenderness over the from this case in bold): greater trochanter is pathognomonic. Relieved with steroid injection Look/ inspection Trendelenburg gait, Feel No tenderness. Measure leg lengths What investigations would you do? for discrepancy (True and apparent)* • X-ray (AP pelvis and lateral X-ray left hip) Move Active Decreased range of motion of left o Joint Space Narrowing hip (all movements) o Subchondral Sclerosis Passive Decreased range of motion of left o Pseudocyst formation hip (in particular medial rotation of o Osteophyte formation hip – stiffness at end of available range) What are the management options? Resisted Nil of note a) Initial management by the GP includes conservative Special tests for this Trendelenburg test* treatments: joint examination Thomas’s test* • Analgesia Other BMI (29); Normal knee examination • Refer for physiotherapy • Promote weight loss if appropriate, What are your differential diagnoses? • Orthopaedic OPD referral (if indicated) • Hip OA b) Orthopaedic team in secondary care: • Knee OA • Non-operative: Analgesia, Physiotherapy, Promote • Nerve root Impingement weight loss if appropriate, Intra-articular injection of • Trochanteric bursitis steroid and local anaesthetic • Operative: Total Hip Replacement (Arthroplasty) Why is Hip OA likely? (Cemented Vs Uncemented; Bearing surfaces (metal, • History: Site of pain left groin and left knee polyethylene, Ceramic); Anterolateral, Direct anterior, • Examination findings: Trendelenburg gait; reduced Posterior approaches) AROM and PROM hip. Positive Thomas’s test left hip

Why are the other differential diagnoses unlikely?

34 35

HIP LABRAL TEAR

History: A 33 year-old Gaelic footballer presents to your General Practice complaining of a 3-month history of right hip and groin pain which is worse when running and jumping and eased with rest. He describes an intermittent snapping feeling over his right hip on certain movements and a locking sensation from time to time. He has no history of trauma and no of note.

What would you look for on examination (clinical findings from this case in bold):

Look/ inspection Antalgic gait Feel Nil of note (no pubic symphysis, greater trochanter or groin tenderness) Move Active Pain/clicking during active range of motion of hip (non-specific); Passive Pain/clicking during passive range of motion of hip (non-specific); Resisted Nil of note Special tests for this Provocative tests: joint examination Anterior labral tear: Pain on movement of hip from a fully flexed, abducted and rotated laterally position to an extended, medially rotated, and adducted position.

Posterior labral tear: Pain on movement of hip from a fully flexed, medially rotated and adducted position to an abducted, lateral rotated and extended position. Other Nil of note

What are your differential diagnoses? • Labral tear of hip • Osteitis Pubis

36

HIP LABRAL TEAR • Trochanteric Bursitis • Osteoarthritis R hip History: • A 33 year-old Gaelic footballer presents to your General Practice complaining of a 3-month history of right hip and Why is labral tear of hip likely? groin pain which is worse when running and jumping and • History: Pain worse with running/ jumping eased with rest. He describes an intermittent snapping feeling • Examination: pain during AROM and PROM hip. over his right hip on certain movements and a locking Positive Anterior Labral Provocative test sensation from time to time. He has no history of trauma and no past medical history of note. Why are other diagnoses are unlikely? • Osteitis Pubis unlikely as: What would you look for on examination (clinical findings o No localised tenderness over the pubic from this case in bold): symphysis • Trochanteric Bursitis unlikely as: Look/ inspection Antalgic gait o No tenderness over the greater trochanter Feel Nil of note (no pubic symphysis, • Osteoarthritis right hip unlikely as: greater trochanter or groin o Relatively young age, painful clicking during tenderness) AROM/PROM not associated classically with Move Active Pain/clicking during active range of osteoarthritis hip motion of hip (non-specific); • Hip dysplasia unlikely as: Passive Pain/clicking during passive range o No previous history of hip problems (from of motion of hip (non-specific); childhood etc.) Resisted Nil of note Special tests for this Provocative tests: What investigations would you do? joint examination Anterior labral tear: Pain on • X-ray: AP Pelvis: to rule-out other causes of movement of hip from a fully flexed, e.g. abducted and rotated laterally position • Hip Dysplasia to an extended, medially rotated, and • Osteoarthritis adducted position. • Acetabular Cysts (can be closely associated with labral tears and osteoarthritis) : Pain on Posterior labral tear • GP: Refer to Orthopaedics with a view to arranging movement of hip from a fully flexed, MRI Arthrogram (effective when looking for acetabular medially rotated and adducted tears) position to an abducted, lateral rotated and extended position. What are the management options? Other Nil of note a) Initial management by GP includes conservative treatments: What are your differential diagnoses? • Rest • Labral tear of hip • Analgesics • Osteitis Pubis • Refer for physiotherapy

36 37

• Orthopaedic referral if problems persist b) Orthopaedic team in secondary care: • Non-operative: Rest, Analgesics, Physiotherapy, Intra-articular steroid injections, appropriate investigations as above (MRI arthrogram may be combined with intra-articular steroid injections) • Operative: Arthroscopic labral debridement

38

• Orthopaedic referral if problems persist DEVELOPMENTAL DYSPLASIA OF THE HIP b) Orthopaedic team in secondary care: • Non-operative: Rest, Analgesics, Physiotherapy, History: Intra-articular steroid injections, appropriate During a baby girl’s six-week check in your General Practice, investigations as above (MRI arthrogram may be her mother expresses concerns about asymmetry of her combined with intra-articular steroid injections) daughter’s thigh folds. This is her first child and she was a • Operative: Arthroscopic labral debridement breech delivery. The baby’s mother informs you that two of her sisters’ children had ‘hip problems’ when they were born.

What would you look for on examination (clinical findings from this case in bold):

Look/ inspection Asymmetric thigh folds Feel Nil of note Move Active Not applicable as examining neonate Passive Not applicable as examining neonate Resisted Not applicable as examining neonate Special tests for this Barlow test positive* joint examination Ortolani Test positive* Other Nil of note

What are your differential diagnoses? • DDH • Traumatic Dislocation

Why is DDH likely? • History: Breech delivery, positive family history, female gender • Examination: Asymmetric thigh folds, Barlow and Ortolani test positive

Why are other diagnoses are unlikely? Traumatic Dislocation of hip is unlikely as: • No history of trauma • Baby is not irritable

What investigations would you do? • Ultrasound (useful until child 3-5 months of age): Evaluates acetabular dysplasia and the presence/absence of hip dislocation

38 39

• X-ray: AP Pelvis (useful when baby older than 5 months): To evaluate hip dislocation and hip dysplasia.

What are the management options? a) GP: Urgent referral to Paediatric Orthopaedic OPD b) Paediatric orthopaedic team in secondary care: • Non-operative: Abduction splinting/ bracing (Pavlik’s harness) o Closed reduction and SPICA casting • Operative: Open reduction and SPICA casting o Open reduction and femoral/pelvic

40

• X-ray: AP Pelvis (useful when baby older than 5 PERTHES DISEASE months): To evaluate hip dislocation and hip dysplasia. History: A mother presents to your General Practice with her 8-year- What are the management options? old son who has had a painless for the past 3 weeks and a) GP: Urgent referral to Paediatric Orthopaedic OPD is now complaining of intermittent left hip and groin pain. He b) Paediatric orthopaedic team in secondary care: denies any history of trauma and has a past medical history of • Non-operative: Abduction splinting/ bracing (Pavlik’s attention deficit hyperactivity disorder (ADHD). harness) o Closed reduction and SPICA casting What would you look for on examination (clinical findings • Operative: Open reduction and SPICA casting from this case in bold): o Open reduction and femoral/pelvic osteotomy Look/ inspection Antalgic gait. Leg length discrepancy is a late sign Feel Nil of note Move Active Pain limited range of motion of left hip (most likely abduction and medial rotation) Passive Pain limited range of motion of left hip (most likely abduction and medial rotation) Resisted Nil of note Special tests for this Trendelenburg test* positive is a late joint examination sign Other Co-operative with examination

What are your differential diagnoses? • Perthes Disease • Slipped Upper Femoral Epiphysis (SUFE) • Septic arthritis • Transient synovitis of hip • Haematological malignancy

Why is Perthes likely? • Male: female ratio Perthes 5:1, classic presentation (painless limp initially), association with ADHD (33% of cases)

Why are other diagnoses unlikely?

40 41

• SUFE is unlikely as: Patients tend to be older and overweight with SUFE. Natural history tends to carry on for a number of months prior to presentation • Septic Arthritis unlikely as: Patients with Septic Arthritis tend to have a more acute presentation, and a history suggesting the child has experienced sudden pain with passive motion and is unable to walk/use limb. Use Kocher score* • Transient Synovitis unlikely as: With Transient Synovitis, patient will complain of acute onset hip pain and be unwilling to use the hip. Usually occurs between the ages of 4 – 8. May be a history of recent viral infection or trauma • Haematological malignancy: Any child presenting with a limp should have a full blood count to out-rule haematological malignancy

What investigations would you do? • X-ray: AP Pelvis and Frog leg lateral views. Check for: • Medial joint space widening • Irregularity of femoral head ossification • Crescent sign (Subchondral fracture) • MRI

What are the management options? GP: Urgent referral to Emergency Department (ED) for further investigations a) Secondary care: From ED, if Perthes diagnosed, referral to Orthopaedic team: Orthopaedicb) approach: • Non-operative: Observation, activity restriction and physiotherapy • Operative: Femoral/Pelvic Osteotomy

42

• SUFE is unlikely as: Patients tend to be older and SLIPPED UPPER FEMORAL EPIPHYSIS overweight with SUFE. Natural history tends to carry on for a number of months prior to presentation History: • Septic Arthritis unlikely as: Patients with Septic A 12-year-old female presents with her father to your General Arthritis tend to have a more acute presentation, fever Practice complaining of left sided insidious hip and groin pain and a history suggesting the child has experienced for the past 2 months. She denies any history of trauma or sudden pain with passive motion and is unable to previous hip problems. walk/use limb. Use Kocher score* • Transient Synovitis unlikely as: With Transient What would you look for on examination (clinical findings Synovitis, patient will complain of acute onset hip pain from this case in bold): and be unwilling to use the hip. Usually occurs between the ages of 4 – 8. May be a history of recent Look/ inspection Raised BMI (34). Antalgic gait. viral infection or trauma Laterally rotated left foot, left • Haematological malignancy: Any child presenting with quadriceps atrophy a limp should have a full blood count to out-rule Feel Nil of note haematological malignancy Move Active Reduced range of motion of left hip (flexion and abduction in particular) What investigations would you do? Passive Reduced range of motion of left hip • X-ray: AP Pelvis and Frog leg lateral views. Check for: (flexion and abduction in • Medial joint space widening particular). • Irregularity of femoral head ossification You may note obligatory lateral • Crescent sign (Subchondral fracture) rotation of left hip on passive • MRI flexion Resisted Nil of note What are the management options? Special tests for this Nil GP: Urgent referral to Emergency Department (ED) for joint examination further investigations Other Normal distal neurovascular status a) Secondary care: From ED, if Perthes diagnosed, referral to Orthopaedic team: What are your differential diagnoses? Orthopaedicb) approach: • Slipped upper femoral Epiphysis (SUFE) • Non-operative: Observation, activity restriction and • Perthes physiotherapy • Hip Fracture • Operative: Femoral/Pelvic Osteotomy Why is SUFE likely? • History: Age appropriate, insidious onset pain, absence of trauma • Examination: Raised BMI, reduced AROM/PROM right hip, obligatory lateral rotation on passive hip flexion

42 43

Why are the other differentials unlikely? • Perthes unlikely as: It normally occurs in young active children (aged 4-8 years), more common in boys than girls (M:F 5:1), generally child will have normal body habitus • Hip Fracture unlikely as: No history of trauma/ malignancy/ use. Pain from fracture is sudden as opposed to insidious in onset

What investigations would you do? GP: Refer Emergency Department (ED) for X-rays/urgent orthopaedic opinion • X-ray: AP Pelvis & frog leg lateral views (to identify subtle slips) • AP Pelvis findings: • Klein’s line - line drawn along superior border femoral neck will not intersect femoral head in a child with SUFE • Epiphysiolysis - Growth plate widening/ lucency • Blurring of proximal femoral metaphysis (due to overlapping of metaphysic and posteriorly displaced epiphysis) • MRI: Indications: Used when radiographs are negative and preslip is suspected • Findings: Epiphyseal widening

What are the management options? ED referral to Orthopaedics for operative management: • • Contralateral prophylactic pinning: Indications - high risk patients (obese, endocrine disorders, initial slip at a younger age)

HIP FRACTURE

History:

44

Why are the other differentials unlikely? You are called to review a 73-year-old widow in the • Perthes unlikely as: It normally occurs in young active emergency department who presented by ambulance after children (aged 4-8 years), more common in boys than falling at home. She describes tripping over a step and feeling girls (M:F 5:1), generally child will have normal body acute onset pain in her right hip and groin on hitting the habitus ground. She has a background history of osteoporosis. • Hip Fracture unlikely as: No history of trauma/ malignancy/ corticosteroid use. Pain from fracture is What would you look for on examination (clinical findings sudden as opposed to insidious in onset from this case in bold):

What investigations would you do? Look/ inspection R leg is shortened and laterally GP: Refer Emergency Department (ED) for X-rays/urgent rotated. Bruising and swelling orthopaedic opinion around right hip • X-ray: AP Pelvis & frog leg lateral views (to identify Feel Tenderness around right hip subtle slips) Move Active Not appropriate as possible hip • AP Pelvis findings: fracture • Klein’s line - line drawn along superior border Passive Not appropriate as possible hip femoral neck will not intersect femoral head in fracture a child with SUFE Resisted Not appropriate as possible hip • Epiphysiolysis - Growth plate widening/ fracture lucency Special tests for this Nil of note • Blurring of proximal femoral metaphysis (due joint examination to overlapping of metaphysic and posteriorly Other Unable to move right hip. Unable to displaced epiphysis) walk. • MRI: Indications: Used when radiographs are negative and preslip is suspected What are your differential diagnoses? • Findings: Epiphyseal widening • Neck of femur fracture right hip • Hip dislocation What are the management options? • Hip osteoarthritis ED referral to Orthopaedics for operative management: • Internal fixation Why is fractured neck of femur likely? • Contralateral prophylactic pinning: Indications - high • History of significant trauma on a background of risk patients (obese, endocrine disorders, initial slip at osteoporosis. Clinical examination findings (in a younger age) particular shortened and laterally rotated affected limb) suggest the diagnosis

Why are the other differentials unlikely? HIP FRACTURE • Hip Dislocation is unlikely as: With a hip dislocation, hip and leg will be in slight flexion, adduction, and History: medial rotation

44 45

• Hip OA is unlikely as: There tends to be a prolonged history with hip OA with a clinical course that lasts from months to years. Patients with hip OA will complain of groin and thigh pain with stiffness and decreased range of motion

What investigations would you do? • X-ray: AP Pelvis • Garden Classification of femoral neck fractures* • CT Scan • Used in comminuted fractures

What are the management options? Emergency department refer patient urgently to Orthopaedic team for review Based on Garden Classification*, options for Orthopaedic team are: a) Non- operative: • Observation, this option is restricted to those who are at high risk for surgical intervention • Review medical reason for fall – address as appropriate • Review bone health – address as appropriate b) Operative: • Open Reduction Internal Fixation: Used in displaced fractures in < 65 • Cannulated Screw: Used in nondisplaced transcervical fractures (Garden I & II) • Hemiarthroplasty: Used in Garden III & IV if predicted survival is < 5years • Total Hip Arthroplasty: Garden III & IV

46

• Hip OA is unlikely as: There tends to be a prolonged AVASCULAR NECROSIS OF THE FEMORAL HEAD history with hip OA with a clinical course that lasts from months to years. Patients with hip OA will History: complain of groin and thigh pain with stiffness and A 60-year-old farmer’s wife presents to your General Practice decreased range of motion complaining of gradual onset left anterior hip pain over the past 6 months. She tells you she is having increasing What investigations would you do? problems with the stairs and walking up hills. On closer • X-ray: AP Pelvis questioning, she admits that her right hip is beginning to • Garden Classification of femoral neck bother her also. You note that she has been taking low dose fractures* prednisolone for the past ten years for idiopathic pulmonary • CT Scan fibrosis. • Used in comminuted fractures What would you look for on examination (clinical findings What are the management options? from this case in bold): Emergency department refer patient urgently to Orthopaedic team for review Look/ inspection Antalgic gait and leg length Based on Garden Classification*, options for Orthopaedic discrepancy* team are: Feel Nil of note a) Non- operative: Move Active Pain limited range of motion left hip • Observation, this option is restricted to those who are greater than right at high risk for surgical intervention Passive Pain limited range of motion left hip • Review medical reason for fall – address as greater than right appropriate Resisted Nil of note • Review bone health – address as appropriate Special tests for this Nil b) Operative: joint examination • Open Reduction Internal Fixation: Used in displaced Other Nil fractures in < 65 • Cannulated Screw: Used in nondisplaced What are your differential diagnoses? transcervical fractures (Garden I & II) • Avascular Necrosis femoral head secondary to • Hemiarthroplasty: Used in Garden III & IV if predicted steroids survival is < 5years • Hip Osteoarthritis • Total Hip Arthroplasty: Garden III & IV • Metastatic disease

Why is avascular necrosis secondary to steroid use likely? • Known side effect of prolonged steroid use (in this case 10 year history of prolonged oral steroid use). Pain limited active and passive range of motion.

Why are the other differentials unlikely?

46 47

• Hip Osteoarthritis: Is a possible diagnosis too but given prolonged oral steroid use in the past, avascular necrosis needs to be top of the list of differentials. • Metastatic/ primary neoplastic disease: Associated with rest pain. If bone metastasis or a primary bone tumour was present, we would also expect night pain, night sweats, fever, .

What investigations would you do? • X-ray: AP Pelvis, frog leg lateral views of affected and Contralateral hip • Steinburg Classification* (Based on X-ray and MRI) • MRI • Used when X-ray is negative and osteonecrosis is still suspected • oedema • Bone Density Scan (DEXA) • If concern for – Bence Jones protein, serum protein electrophoresis (SPEP) for , hypercalcaemia, anaemia, thrombocytopenia are non specific markers

What are the management options? a) GP: Once clinical suspicion of avascular necrosis, likely refer patient in to Emergency Department for urgent orthopaedic review. b) Emergency department refer patient urgently to Orthopaedic team for review. c) Orthopaedic team options: i) Non-operative: • Bisphosphonates: Indicated for pre-collapse vascular necrosis ii) Operative: • Multiple options present, decided on a case-by-case basis depending on the extent and region of osteonecrosis. • Core decompression +/- (early AVN, stimulates healing via angiogenesis)

48

• Hip Osteoarthritis: Is a possible diagnosis too but • Total Hip replacement (younger patients, Patients >65 given prolonged oral steroid use in the past, avascular with large lesions) necrosis needs to be top of the list of differentials. • Total (contraindicated in AVN • Metastatic/ primary neoplastic disease: Associated secondary to long term steroid use) with rest pain. If bone metastasis or a primary bone • Hip arthrodesis (very young patient in labour intensive tumour was present, we would also expect night pain, occupation) night sweats, fever, malaise.

What investigations would you do? • X-ray: AP Pelvis, frog leg lateral views of affected and Contralateral hip • Steinburg Classification* (Based on X-ray and MRI) • MRI • Used when X-ray is negative and osteonecrosis is still suspected • Bone marrow oedema • Bone Density Scan (DEXA) • If concern for neoplasm – Bence Jones protein, serum protein electrophoresis (SPEP) for multiple myeloma, hypercalcaemia, anaemia, thrombocytopenia are non specific markers

What are the management options? a) GP: Once clinical suspicion of avascular necrosis, likely refer patient in to Emergency Department for urgent orthopaedic review. b) Emergency department refer patient urgently to Orthopaedic team for review. c) Orthopaedic team options: i) Non-operative: • Bisphosphonates: Indicated for pre-collapse vascular necrosis ii) Operative: • Multiple options present, decided on a case-by-case basis depending on the extent and region of osteonecrosis. • Core decompression +/- Bone grafting (early AVN, stimulates healing via angiogenesis)

48 49

TRANSIENT SYNOVITIS OF THE HIP

History: A mother brings her 4-year-old son to your General Practice for review. He has been complaining of fluctuating right hip pain over the past 24 hours and refuses to weight bear. She informs you that the pain came on yesterday morning, eased during the day yesterday to the point where he was able to walk with a limp but was ‘very bad again this morning’. Of note the child has had no history of recent infections and is well otherwise.

What would you look for on examination (clinical findings from this case in bold):

Look/ inspection Child in a buggy. Refusing to stand. Hip flexed, abducted and laterally rotated (position of least intracapsular pressure) Feel Nil of note Move Active Child will be unwilling to move their hip and weight bear Passive The child has moderate restriction of hip abduction, there is full pain free active range of motion of lumbar spine and ipsilateral knee (on the same side). Resisted Nil of note Special tests for this Nil of note joint examination Other Apyrexial.

What are your differential diagnoses? • Transient Synovitis of Hip • Septic Arthritis • Perthes disease • Slipped Upper Femoral Epiphysis

Why is transient synovitis of hip likely?

50

TRANSIENT SYNOVITIS OF THE HIP • History: Sudden onset, fluctuating symptoms which ease as day goes on History: • Examination: Well, apyrexial, mild restriction of hip A mother brings her 4-year-old son to your General Practice abduction on passive range of motion for review. He has been complaining of fluctuating right hip pain over the past 24 hours and refuses to weight bear. She Why are the other differentials unlikely? informs you that the pain came on yesterday morning, eased • Septic Arthritis unlikely as: The child would appear during the day yesterday to the point where he was able to more unwell and pyrexial. Do need to rule this out walk with a limp but was ‘very bad again this morning’. Of note though with bloods FBC, ESR, CRP the child has had no history of recent infections and is well • Perthes: The pain is unlikely to ease during the day in otherwise. Perthes • Slipped Upper Femoral Epiphysis (SUFE): Patients What would you look for on examination (clinical findings tend to be older and more overweight with SUFE. The from this case in bold): natural history of SUFE tends to carry on for a number of months prior to presentation Look/ inspection Child in a buggy. Refusing to stand. Hip flexed, abducted and laterally What investigations would you do? rotated (position of least • Blood tests: FBC, ESR, CRP intracapsular pressure) • Use Kocher Criteria for Septic Arthritis* Feel Nil of note • FBC will help rule out haematological Move Active Child will be unwilling to move their hip malignancy and weight bear • X-rays: Passive The child has moderate restriction of • AP Pelvis and frog leg views hip abduction, there is full pain free • Normal Appearance active range of motion of lumbar spine • May show joint space widening medially and ipsilateral knee (on the same • Ultrasound: side). • Indicated if suspected infection Resisted Nil of note • Used to detect intra-capsular fluid/ synovial Special tests for this Nil of note thickening joint examination • Difficult to differentiate between Transient Other Apyrexial. synovitis and Septic Arthritis

• MRI What are your differential diagnoses? • Can help differentiate between septic arthritis • Transient Synovitis of Hip and transient synovitis • Septic Arthritis st • May require general anaesthetic and is not 1 • Perthes disease line investigation • Slipped Upper Femoral Epiphysis What are the management options? Why is transient synovitis of hip likely?

50 51 a) GP: Refer patient to Emergency Department for review and further investigations (see above). Once transient synovitis diagnosed, depending on severity, either discharge home or refer to Orthopaedics for review b) Orthopaedic secondary care management: • Non-operative: NSAIDS, rest and close observation • (If low clinical suspicion for septic arthritis, Kocher score <2, symptoms improve/resolve with NSAIDs) • No operative treatment recommended for transient synovitis of hip

52 a) GP: Refer patient to Emergency Department for review and SEPTIC ARTHRITIS OF THE HIP further investigations (see above). Once transient synovitis diagnosed, depending on severity, either discharge home or History: refer to Orthopaedics for review A 6-year-old boy presents to the Emergency Department with b) Orthopaedic secondary care management: his parents, as he has been unable to walk properly for 3 days • Non-operative: NSAIDS, rest and close observation due to a painful left hip. He has stopped playing with his • (If low clinical suspicion for septic arthritis, Kocher brothers at home and he was unwell yesterday, with a fever score <2, symptoms improve/resolve with NSAIDs) and one episode of vomiting. He has no past medical history • No operative treatment recommended for transient of note and lives with both parents and two older brothers. synovitis of hip What would you look for on examination (clinical findings from this case in bold):

Look/ inspection Looks unwell. Antalgic gait. Redness, swelling, left hip in FABER position (Flexed, abducted and laterally rotated) Feel Tenderness and warmth over lateral aspect left hip Move Active Painful active range of motion of left hip Passive Painful passive range of motion of left hip Resisted No pain on resisted movement testing around left hip Special tests for this Nil joint examination Other Pyrexial (temperature 38˚Centigrade), BMI normal

What are your differential diagnoses? • Septic Arthritis • Transient Synovitis (no infective symptoms, no fever) • Perthes disease (no infective symptoms associated)

Why is septic arthritis likely? • Acute onset pain, associated fever and feeling unwell. Pain limited AROM and PROM left hip.

Why is Transient Synovitis unlikely?

52 53

• With irritable hip, patient would be otherwise well

Why is Perthes disease unlikely? • With Perthes disease, there would be no fever, no redness and no swelling around left hip

What investigations would you do? • Bloods: WCC, ESR, CRP (all raised) • X-ray: (AP and lateral of the joint in question) may show joint space widening or effusion • Ultrasound: Can assess for a joint effusion in a large joint such as the hip, and can also be helpful to guide aspirations • MRI: Can detect joint effusions or adjacent bone involvement (e.g. ) • Joint aspiration for microscopy culture and sensitivity. Also send for crystal analysis

What are the management options in the emergency department? 1. Urgent referral to Orthopaedics: 2. Non-operative management: IV antibiotics and analgesia 3. Operative management: • Hip aspiration and IV antibiotics • High clinical suspicion of septic arthritis • If symptoms fail to resolve with NSAIDS • Systemic Infection • Kocher score >2 • Irrigation and debridement of hip • Documented Infection • Kocher score 4/4 • Severe systemic infection with suspected septic hip

54

• With irritable hip, patient would be otherwise well KNEE OSTEOARTHRITIS

Why is Perthes disease unlikely? History: • With Perthes disease, there would be no fever, no A 62-year-old female presents to your General Practice redness and no swelling around left hip complaining of right anterior knee pain for the past 18 months. The knee pain is now limiting her mobility as she struggles to What investigations would you do? walk more than 300 metres without pain and finds descending • Bloods: WCC, ESR, CRP (all raised) stairs difficult. The pain is no longer responsive to • X-ray: (AP and lateral of the joint in question) may Paracetamol or NSAIDs, and it keeps her awake at night. She show joint space widening or effusion denies any history of trauma or previous knee problems. • Ultrasound: Can assess for a joint effusion in a large joint such as the hip, and can also be helpful to guide What would you look for on examination (clinical findings aspirations from this case in bold): • MRI: Can detect joint effusions or adjacent bone involvement (e.g. osteomyelitis) Look/ inspection Antalgic gait • Joint aspiration for microscopy culture and sensitivity. Fixed flexion Deformity Right knee Also send for crystal analysis Quadriceps Atrophy right knee (especially Vastus medialis) What are the management options in the emergency Valgus or Varus alignment department? Feel Joint line tenderness 1. Urgent referral to Orthopaedics: Effusion 2. Non-operative management: IV antibiotics and analgesia Crepitus 3. Operative management: Patellar Tap test* for effusion • Hip aspiration and IV antibiotics Move Active Painful and decreased range of • High clinical suspicion of septic arthritis motion of right knee with or without • If symptoms fail to resolve with NSAIDS flexion deformity • Systemic Infection Passive Painful and decreased range of • Kocher score >2 motion of right knee with or without • Irrigation and debridement of hip flexion deformity Crepitus • Documented Infection Resisted Nil of note • Kocher score 4/4 Special tests for this a) Cruciate ligament examination: • Severe systemic infection with suspected joint examination - Anterior Drawer Test*: negative septic hip - Lachman’s Test*: negative b) Medial and lateral collateral ligaments: Valgus and varus stress tests*: negative c) Meniscus examination: - McMurray’s test*: negative

54 55

Other Examine hip and ankle (both normal in this case) Check BMI (BMI is 34kg/m2) Distal neurovascular examination (normal)

What are your differential diagnoses? • Knee OA • Knee Pain Secondary to Hip Pathology • Medial • Inflammatory Arthritis

Why is Knee OA likely? • Decreased active and passive range of motion of knee joint with crepitus, a 10 degrees fixed flexion deformity is present and there is joint-line tenderness

Why are the other diagnoses unlikely? • Hip Pathology unlikely: Patient has normal examination right hip (in particular no stiffness on passive medial rotation testing right hip) • Medial Meniscal Tear unlikely: No history of trauma, gradual increase in severity of symptoms, reduced range of motion. Anterior knee pain is suggestive of OA in patello-femoral joint • Inflammatory Arthritis unlikely: No early morning stiffness, no bilateral symmetrical polyarticular of joints, no family history of inflammatory arthritis. Well otherwise. Could send off bloods checking arthritis screen if suspicious e.g. ESR, CRP, Rheumatoid factor, anti-CCP antibodies, ANCA

What investigations would you do? • X-ray knee: AP, lateral & Skyline views of Right Knee, Expected findings: • Joint Space Narrowing • Subchondral Sclerosis • Pseudocyst formation • Osteophyte formation

56

Other Examine hip and ankle (both normal in • On lateral view – Evidence of above for this case) patella-femoral joint 2 Check BMI (BMI is 34kg/m ) • On Lateral view – Loose bodies in posterior Distal neurovascular examination capsule of knee (Causing FFD) (normal) • Skyline view – AP X-ray of patello-femoral Joint What are your differential diagnoses? • Consider X-ray Pelvis (AP) to assess hip joint if any • Knee OA symptoms or signs of hip pathology. • Knee Pain Secondary to Hip Pathology • MRI Knee: Small role for MRI in degenerative knee • Medial Meniscus Tear disease. X-rays usually give all the information • Inflammatory Arthritis required

Why is Knee OA likely? What are the management options? • Decreased active and passive range of motion of a) Initial management by the GP includes conservative knee joint with crepitus, a 10 degrees fixed flexion treatments: deformity is present and there is joint-line tenderness • Encourage weight loss if overweight • Refer for physiotherapy Why are the other diagnoses unlikely? • Analgesia: Start with Paracetamol & NSAIDs (topical • Hip Pathology unlikely: Patient has normal NSAIDs if possible) as per WHO pain ladder. examination right hip (in particular no stiffness on • Intra-articular Injection of Steroid and Local passive medial rotation testing right hip) Anaesthetic • Medial Meniscal Tear unlikely: No history of trauma, • Orthopaedic referral if appropriate gradual increase in severity of symptoms, reduced b) Orthopaedic secondary care management: range of motion. Anterior knee pain is suggestive of • Non-operative: Analgesia, Physiotherapy, Intra- OA in patello-femoral joint articular steroid injections • Inflammatory Arthritis unlikely: No early morning • Operative: stiffness, no bilateral symmetrical polyarticular i. Total Knee Arthroplasty (Replacement) inflammation of joints, no family history of ii. Unicompartmental Arthroplasty inflammatory arthritis. Well otherwise. Could send off iii. bloods checking arthritis screen if suspicious e.g. ESR, CRP, Rheumatoid factor, anti-CCP antibodies, ANCA

What investigations would you do? • X-ray knee: AP, lateral & Skyline views of Right Knee, Expected findings: • Joint Space Narrowing • Subchondral Sclerosis • Pseudocyst formation • Osteophyte formation

56 57

ANTERIOR CRUCIATE LIGAMENT RUPTURE

History: A 21-year-old Gaelic footballer presents to his GP 4 days after injuring his right knee. He describes getting his studs caught in the pitch as he attempted to turn after catching a ball. He felt a ‘pop’. He was unable to play on and was in significant pain. The knee was very swollen in the immediate aftermath. He has been limping and lacks confidence in the stability of his knee while walking

What would you look for on examination (clinical findings from this case in bold):

Look/ inspection Antalgic gait Swelling of right knee joint Feel Patellar tap test for effusion (large effusion present) Right knee joint line tenderness (ACL rupture can be associated with meniscus injury) Move Active Decreased ROM right knee due to swelling Passive Decreased ROM right knee due to swelling Resisted Nil of note

58

ANTERIOR CRUCIATE LIGAMENT RUPTURE Special tests for this a) Cruciate ligament examination: joint examination - Anterior Drawer Test*: positive History: - Lachman’s Test*: positive A 21-year-old Gaelic footballer presents to his GP 4 days after - Pivot Shift Test - Students not injuring his right knee. He describes getting his studs caught expected to elicit this complex clinical in the pitch as he attempted to turn after catching a ball. He test but to know it exists as a further felt a ‘pop’. He was unable to play on and was in significant means of demonstrating knee pain. The knee was very swollen in the immediate aftermath. instability. He has been limping and lacks confidence in the stability of b) Medial and lateral collateral his knee while walking ligaments: - Valgus and varus stress tests*: What would you look for on examination (clinical findings Normal from this case in bold): c) Meniscus examination: - McMurray’s test*: Too painful to Look/ inspection Antalgic gait perform in the acute setting. Swelling of right knee joint Other Nil of note Feel Patellar tap test for effusion (large effusion present) What are you differential diagnoses? Right knee joint line tenderness • ACL Rupture (ACL rupture can be associated • PCL Rupture with meniscus injury) • / Meniscus injury Move Active Decreased ROM right knee due to swelling Why is an ACL rupture likely? Passive Decreased ROM right knee due to • Typical history of a traumatic twisting sports injury, swelling with a ‘pop’ and immediate swelling and pain. Positive Resisted Nil of note Anterior Drawer Test/ Lachmanns test

Why are the other diagnoses unlikely? • PCL rupture unlikely: Much less frequent occurrence than ACL, mechanism is usually a fall or blow to a flexed knee, would suspect a positive posterior drawer test on examination • Cartilage or Meniscus injury unlikely: Effusion usually presents much later (24hrs), symptoms of locking rather than instability

What investigations would you do? • MRI Knee: • Gold standard investigation with high sensitivity and specificity for diagnosing ACL

58 59

ruptures (and associated concurrent e.g. O’Donoghue’s triad of ACL, MCL and medial meniscus injury) • X-ray Knee • To look for a fracture • Occasionally bony avulsion of ACL from tibia evident seen on X-ray in adolescents

What are the management options? As a GP you refer immediately to orthopaedic surgery as a ruptured ACL is queried a) Initial management by the GP includes conservative treatments: • Rest, Ice, Elevation • Analgesia: start with Paracetamol & NSAIDS. Further analgesia as required in accordance with WHO Pain Ladder • Refer for physiotherapy – may include specific focus on quadriceps strengthening and proprioception training to compensate for instability b) Orthopaedic secondary care management: • Non-operative: Analgesia, Physiotherapy, Functional bracing (Don Joy brace) • Operative: Arthroscopic ACL reconstruction – Patellar Tendon or Hamstring Tendon Graft reconstruction

60

ruptures (and associated concurrent injuries POSTERIOR CRUCIATE LIGAMENT RUPTURE e.g. O’Donoghue’s triad of ACL, MCL and medial meniscus injury) History: • X-ray Knee A 28-year-old rugby player presents to her GP 3 weeks after • To look for a fracture injuring her knee. She describes getting tackled from behind • Occasionally bony avulsion of ACL from tibia and landing forcefully onto her flexed right knee. She felt a evident seen on X-ray in adolescents ‘pop’. She was unable to play on. The knee was very swollen in the immediate aftermath. She has difficulty descending What are the management options? stairs and feels the knee will give way if she takes more than As a GP you refer immediately to orthopaedic surgery as a one step at a time. She attempted a return to light jogging ruptured ACL is queried yesterday but the knee felt “not quite right”. a) Initial management by the GP includes conservative treatments: What would you look for on examination (clinical findings • Rest, Ice, Elevation from this case in bold): • Analgesia: start with Paracetamol & NSAIDS. Further analgesia as required in accordance with WHO Pain Look/ inspection Antalgic gait Ladder Effusion • Refer for physiotherapy – may include specific focus Affected leg sags posteriorly when on quadriceps strengthening and proprioception examined from the side with knees training to compensate for instability flexed b) Orthopaedic secondary care management: Feel Patellar tap test* for Effusion • Non-operative: Analgesia, Physiotherapy, Functional Move Active Decreased ROM right knee due to bracing (Don Joy brace) swelling in early stages • Operative: Arthroscopic ACL reconstruction – Patellar Passive Decreased ROM right knee due to Tendon or Hamstring Tendon Graft reconstruction swelling in early stages Resisted Nil of note

60 61

Special tests for this a) Cruciate ligament examination: joint examination - Posterior Drawer Test* positive: - Anterior Drawer Test* negative. - Lachmann’s Test*: negative - Pivot Shift Test - Students not expected to elicit this complex clinical test but to know it exists as a further means of demonstrating knee instability. b) Medial and lateral collateral ligaments: - Valgus and varus stress tests*: Normal c) Meniscus examination: - McMurray’s test*: Too painful to perform in the acute setting. Other Nil of note

What are your differential diagnoses? • PCL Rupture • ACL Rupture • Cartilage / Meniscus injury

Why is a PCL rupture likely? • Unstable knee with positive posterior drawer test

Why are the other diagnoses unlikely? • ACL rupture unlikely: Would have positive anterior drawer test/pivot shift test/ Lachmanns • Cartilage or Meniscus injury unlikely: As there are no symptoms of locking (more associated with cartilage injury than instability)

What investigations would you do? • X-ray: Right Knee • Out-rule fracture • Lateral X-ray can reveal a sag • MRI Right Knee:

62

Special tests for this a) Cruciate ligament examination: • Gold standard investigation with high joint examination - Posterior Drawer Test* positive: sensitivity and specificity for diagnosing PCL - Anterior Drawer Test* negative. ruptures - Lachmann’s Test*: negative - Pivot Shift Test - Students not What are the management options? expected to elicit this complex clinical a) Initial management by the GP includes conservative test but to know it exists as a further treatments: means of demonstrating knee • Rest, Ice, Elevation instability. • Analgesia: Start with Paracetamol & NSAIDS. Further b) Medial and lateral collateral analgesia as required in accordance with WHO Pain ligaments: Ladder - Valgus and varus stress tests*: • Refer for physiotherapy – may include specific focus Normal on quadriceps strengthening and proprioception c) Meniscus examination: training to compensate for instability - McMurray’s test*: Too painful to b) Refer for orthopaedic assessment as may need operative perform in the acute setting. management. Orthopaedic secondary care management: Other Nil of note • Non-operative: Analgesia, Physiotherapy, Functional bracing (Don Joy brace) What are your differential diagnoses? • Operative: Treatment usually non-operative but if on- • PCL Rupture going instability consider PCL reconstruction • ACL Rupture • Cartilage / Meniscus injury

Why is a PCL rupture likely? • Unstable knee with positive posterior drawer test

Why are the other diagnoses unlikely? • ACL rupture unlikely: Would have positive anterior drawer test/pivot shift test/ Lachmanns • Cartilage or Meniscus injury unlikely: As there are no symptoms of locking (more associated with cartilage injury than instability)

What investigations would you do? • X-ray: Right Knee • Out-rule fracture • Lateral X-ray can reveal a sag • MRI Right Knee:

62 63

MEDIAL COLLATERAL LIGAMENT INJURY

History: A 34-year-old teacher presents to her GP one day after injuring her left knee. She describes slipping while hill-walking yesterday. She felt a ‘pop’ on the inside of her left knee. She was able to walk down the mountain but had some pain over this area. The knee was stiff and a bit swollen since she woke up today.

What would you look for on examination (clinical findings from this case in bold):

Look/ inspection Antalgic gait Feel Tenderness (above or below left knee joint line) a. Tenderness should be proximal or distal to the joint line for a MCL In jury b. Joint line tenderness suggests meniscus injury Move Active Full ROM left knee Passive Full ROM left knee Resisted Nil of note Special tests for a) Medial and lateral collateral this joint ligaments tested with a Varus examination and *: negative b) Cruciate ligament examination: - Posterior Drawer Test*: Negative - Anterior Drawer Test*: Negative. c) Meniscus examination: - McMurray’s test*: Negative, may be too painful to perform in the acute setting Other Nil of note

64

MEDIAL COLLATERAL LIGAMENT INJURY What are your differential diagnoses? • MCL Injury left knee History: • ACL Rupture A 34-year-old teacher presents to her GP one day after • Cartilage / Meniscus injury injuring her left knee. She describes slipping while hill-walking yesterday. She felt a ‘pop’ on the inside of her left knee. She Why is a MCL injury likely? was able to walk down the mountain but had some pain over • Typical history. Full ROM is still present. Positive this area. The knee was stiff and a bit swollen since she woke valgus stress test. Tender above or below joint line up today. Why are the other diagnoses unlikely? What would you look for on examination (clinical findings • ACL rupture unlikely as: Patient will describe from this case in bold): immediate effusion / haemarthrosis. anterior drawer test and Lachmann test findings are negative on Look/ inspection Antalgic gait examination Feel Tenderness (above or below • Cartilage or Meniscus injury unlikely as: There are no left knee joint line) symptoms from within the knee joint and no a. Tenderness should be symptoms of locking proximal or distal to the joint line for a MCL In jury What investigations would you do? b. Joint line tenderness In the GP practice you could consider no investigations if a suggests meniscus injury mild ligament sprain, but you could consider the following Move Active Full ROM left knee investigations if moderate or severe sprain: Passive Full ROM left knee • X-ray: Knee Resisted Nil of note o Outrule fracture Special tests for a) Medial and lateral collateral • MRI Knee: this joint ligaments tested with a Varus o Differentiates between Grade 1 and 2 MCL examination and Valgus Stress Test*: sprains and Grade 3 ruptures of the MCL negative while also accurately diagnosing any intra- b) Cruciate ligament articular injuries examination: - Posterior Drawer Test*: What are the management options? Negative a) Initial management by the GP includes conservative - Anterior Drawer Test*: treatments: Negative. • Rest, Ice, Elevation c) Meniscus examination: • Analgesia: Start with Paracetamol & NSAIDS. Further - McMurray’s test*: Negative, analgesia as required in accordance with WHO Pain may be too painful to perform in Ladder the acute setting • Refer for physiotherapy Other Nil of note • Unlikely to require orthopaedic referral given history and examination findings

64 65 b) Orthopaedic secondary care management: • Non-operative: Analgesia, Physiotherapy, Functional bracing (Don Joy brace) • Operative: Grade 3 (complete) MCL ruptures sometimes require operative fixation/ reconstruction

66 b) Orthopaedic secondary care management: MENISCAL KNEE INJURY • Non-operative: Analgesia, Physiotherapy, Functional bracing (Don Joy brace) History: • Operative: Grade 3 (complete) MCL ruptures A 21-year-old student presents to her GP 2 days after injuring sometimes require operative fixation/ reconstruction her left knee. She was playing hockey, collided with another player and twisted on her left knee. There was no pop, but she was unable to play on. The knee became swollen the following day and she has been limping since. She has noticed a clicking sensation in her knee and it felt like it ‘locked’ twice, but when she shook her knee this locking sensation went away.

What would you look for on examination (clinical findings from this case in bold):

Look/ inspection Antalgic gait Swelling of joint Feel Patellar tap test for effusion (large effusion present) Left knee joint line tenderness Move Active Decreased ROM left knee due to swelling in early stages Passive Decreased ROM left knee due to swelling in early stages Resisted Nil of note Special tests for this joint a) Meniscus examination: examination - McMurray’s test*: Positive. b) Cruciate ligament examination: - Posterior Drawer Test*: Negative - Anterior Drawer Test*: Negative. c) Medial and lateral collateral ligaments: - Valgus and varus stress tests*: Normal Other Nil of note

What are your differential diagnoses? • Cartilage/ Meniscus injury

66 67

• ACL Rupture • PCL Rupture

Why is a meniscal injury likely? • Typical history of locking and an effusion developing a day after the incident. Positive McMurray’s test

Why are other diagnoses are unlikely? • ACL rupture unlikely: No history of a ‘pop’ or immediate swelling. Negative Anterior Drawer Test • PCL rupture unlikely: Much less frequent occurrence than ACL, mechanism is usually a fall or blow to a flexed knee, would suspect a positive posterior drawer test on examination

What investigations would you do? There may be no need to investigate if there is a typical history and the patient is mobile • X-ray Knee § Should be normal in young person with meniscal injury • MRI Knee: § Tear in meniscus can be shown

What are the management options? a) Initial management by the GP includes conservative treatments: • Rest, Ice, Elevation • Analgesia: Start with Paracetamol & NSAIDS. Further analgesia as required in accordance with WHO Pain Ladder • Refer for physiotherapy • Referral to orthopaedic team if persistent symptoms b) Orthopaedic secondary care management: • Non-operative: Analgesia, Physiotherapy • Operative: Arthroscopy. Partial meniscectomy, Meniscal repair

68

• ACL Rupture DEEP VEIN THROMBOSIS POST TOTAL HIP • PCL Rupture REPLACEMENT

Why is a meniscal injury likely? History: • Typical history of locking and an effusion developing a You are called to review a 68 year old retired army officer in day after the incident. Positive McMurray’s test the emergency department who has presented complaining of right lower leg pain and swelling over the past 2 days. Of note Why are other diagnoses are unlikely? he is 3 weeks post right total hip replacement (THR). He • ACL rupture unlikely: No history of a ‘pop’ or informs you that he has found it very difficult to mobilise since immediate swelling. Negative Anterior Drawer Test the operation due to poorly controlled pain. • PCL rupture unlikely: Much less frequent occurrence than ACL, mechanism is usually a fall or blow to a What would you look for on examination (clinical findings flexed knee, would suspect a positive posterior drawer from this case in bold): test on examination Look/ inspection Antalgic gait, red and swollen right What investigations would you do? lower leg. Healing right THR scar. There may be no need to investigate if there is a typical No obvious redness/discharge from history and the patient is mobile scar. • X-ray Knee Feel Tender and warm right calf § Should be normal in young person with Pitting oedema right lower leg meniscal injury Move Active Full range of motion right ankle and • MRI Knee: knee but pain during movement § Tear in meniscus can be shown Passive Full range of motion right ankle and knee but pain during movement What are the management options? Resisted No pain reproduced on resisted a) Initial management by the GP includes conservative movement right knee/ankle treatments: Special tests for this Nil • Rest, Ice, Elevation joint examination (Homan’s sign is not commonly used • Analgesia: Start with Paracetamol & NSAIDS. Further and lacks sensitivity and specificity for analgesia as required in accordance with WHO Pain a DVT. Ladder Other Apyrexial. Check mid-calf • Refer for physiotherapy circumference on both lower limbs • Referral to orthopaedic team if persistent symptoms (affected side wider girth compared b) Orthopaedic secondary care management: to unaffected side if DVT present). • Non-operative: Analgesia, Physiotherapy • Operative: Arthroscopy. Partial meniscectomy, What are your differential diagnoses? Meniscal repair • Deep vein thrombosis (DVT) • Cellulitis • Ruptured Baker’s Cyst • Right calf strain

68 69

Why is a DVT of right lower leg likely? • History: 3 weeks post right THR, acute onset pain and swelling right lower leg. • Clinical examination – right lower leg red, warm, tender, pitting oedema, more swollen than left lower leg.

Why are the other differentials unlikely? • Cellulitis unlikely as: There is no local wound or obvious sign of entry of infection, no fever. Need to check bloods (FBC, ESR, CRP) if diagnostic doubt • Ruptured Bakers Cyst unlikely as: With this, pain is localised to medial/ lateral joint line and it can be associated with locking (no history of same). A popliteal mass is visible and palpable on extension of knee with a ruptured bakers cyst (not in this case) • Right calf strain unlikely as: There is no pain on resisted movement

What investigations would you do? • Bloods: FBC (would expect normal white cell count with DVT (can be raised with infection e.g. cellulitis); CRP (can be raised with inflammation), D-dimers (sensitive but not specific - good at identifying those patients with the disease but not as good as identifying those patients without the disease) • Calculate Wells score* • Venous Duplex Ultrasound: Gold-standard

What are the management options? Once DVT confirmed: • Low molecular weight heparin e.g. Enoxaparin 1.5mg/kg/24hrs subcutaneously once daily (or until adequate oral anticoagulation established) • Warfarin (as per International Normalised Ratio (INR)) for at least 3 months (6 months if no cause found/recurrent DVT/ thrombophilia) • Novel Oral Anticoagulants (NOACs) can be considered for those patients intolerant of needles or

70

not suitable for warfarin therapy/monitoring. Why is a DVT of right lower leg likely? Disadvantage is limited experience with these • History: 3 weeks post right THR, acute onset pain and . swelling right lower leg. • Inferior vena cava filters may be used if • Clinical examination – right lower leg red, warm, anticoagulation has to be stopped e.g. active bleeding tender, pitting oedema, more swollen than left lower or if it fails (filter helps to minimise risk of Pulmonary leg. Embolism (PE)).

Why are the other differentials unlikely? • Cellulitis unlikely as: There is no local wound or obvious sign of entry of infection, no fever. Need to check bloods (FBC, ESR, CRP) if diagnostic doubt • Ruptured Bakers Cyst unlikely as: With this, pain is localised to medial/ lateral joint line and it can be associated with locking (no history of same). A popliteal mass is visible and palpable on extension of knee with a ruptured bakers cyst (not in this case) • Right calf strain unlikely as: There is no pain on resisted movement

What investigations would you do? • Bloods: FBC (would expect normal white cell count with DVT (can be raised with infection e.g. cellulitis); CRP (can be raised with inflammation), D-dimers (sensitive but not specific - good at identifying those patients with the disease but not as good as identifying those patients without the disease) • Calculate Wells score* • Venous Duplex Ultrasound: Gold-standard

What are the management options? Once DVT confirmed: • Low molecular weight heparin e.g. Enoxaparin 1.5mg/kg/24hrs subcutaneously once daily (or until adequate oral anticoagulation established) • Warfarin (as per International Normalised Ratio (INR)) for at least 3 months (6 months if no cause found/recurrent DVT/ thrombophilia) • Novel Oral Anticoagulants (NOACs) can be considered for those patients intolerant of needles or

70 71

SPINE

72

EXAMPLES OF SPINAL EXAMINATIONS SPINE Cervical spine examination

Look: • Skin: Erythema, scars • Soft Tissue: Swelling, bruising, symmetry, muscle wasting (look at cervical spine and upper limbs) • Bone: Deformity • Posture: e.g. stiff posture with loss of cervical lordosis (e.g. after whiplash injury), e.g. protracted posture (chin and shoulders) due to upper thoracic spine kyphosis

Feel: • Skin: Temperature • Soft Tissue: Paraspinal tenderness, tenderness upper fibres Trapezius • Bone: Spinal tenderness e.g. check from base of occiput to vertebra prominens (7th cervical vertebra)

Move: • Range of Motion – Active – Cervical Spine Flexion, Extension, Right Rotation, Left Rotation, Right Lateral Flexion, Left Lateral Flexion • During spinal examination, medical students are only expected to perform Active Movement. • Neither Passive Movements nor Resisted Movements are routinely performed. This may seem counter- intuitive after everything you have been taught to date (e.g. how will you then know if the problem is arthrogenic or myogenic?). The examination of the Cervical and Lumbar spines are both relatively non- specific in this regard.

Other: • Neurological examination of both upper limbs is typically part of the cervical spine examination.

72 73

Thoracic and lumbar spine examination

Look: • Skin: Erythema, scars • Soft Tissue: Swelling, bruising, rash, muscle wasting, hairy patch covering congenital abnormality • Bone: Rib hump, abnormal curvature (e.g. scoliosis)

Feel: • Skin: Temperature • Soft Tissue: Tenderness paraspinal muscles thoracic/lumbar spine • Bone: Mid-line spinous processes from 1st thoracic vertebra to 5th lumbar vertebra – check for bone alignment and tenderness. Iliac crests are level with the 4th/ 5th Lumbar intervertebral space. of lumbar spine although done in video is not often done in practice so can be left out.

Move: Active and Passive movement • Range of Motion – Active • Thoracic Spine – Rotation to right and left sides • Lumbar Spine – Flexion, Extension and Lateral Flexion to each side.

(Resisted movement are not performed with spinal examination)

Special tests:

• Schober’s Test: o Assesses movement in the lower lumbar spine. The examiner places a finger (or mark) on the patient’s skin centrally over the firfh lumbar vertebra. Two other marks are made, one 10cm above this point and one 5cm below this point. There are therefore two marks, 15cm apart.

74

Thoracic and lumbar spine examination o The patient is asked to bend forwards to touch the floor. If the two points to do not Look: separate by more than 5cm (over 20cm in • Skin: Erythema, scars total) this is a positive Schober’s Test. This • Soft Tissue: Swelling, bruising, rash, muscle wasting, may indicate restriction of lumbar spine hairy patch covering congenital abnormality flexion, e.g. can be associated with • Bone: Rib hump, abnormal curvature (e.g. scoliosis) ankylosing spondylitis.

Feel: • (SLR) • Skin: Temperature o The examiner lifts each of the patient’s • Soft Tissue: Tenderness paraspinal muscles outstretched legs off the bed to assess if pain thoracic/lumbar spine is reproduced in the distribution of the sciatic • Bone: Mid-line spinous processes from 1st thoracic nerve (with the patient supine and their knees vertebra to 5th lumbar vertebra – check for bone extended). alignment and tenderness. Iliac crests are level with o If pain is reproduced (e.g. shooting pain down the 4th/ 5th Lumbar intervertebral space. Percussion of the back of the leg), this may indicate a lumbar spine although done in video is not often done ‘radiculopathy’ or possible impingement of the in practice so can be left out. .

Move: • Wall test Active and Passive movement o The patient is asked to stand against a flat • Range of Motion – Active wall. The examiner tries to slide their hand between the lumbar spine and the wall. If • Thoracic Spine – Rotation to right and left sides there is excessive lumbar lordosis, the entire • Lumbar Spine – Flexion, Extension and Lateral hand or arm may be able to fit in this space. Flexion to each side.

Other: (Resisted movement are not performed with spinal • Neurological examination of both lower limbs is examination) typically part of the cervical spine examination.

Special tests:

• Schober’s Test: Assesses movement in the lower lumbar o spine. The examiner places a finger (or mark) on the patient’s skin centrally over the firfh lumbar vertebra. Two other marks are made, one 10cm above this point and one 5cm below this point. There are therefore two marks, 15cm apart.

74 75

CERVICAL RADICULOPATHY

History: 62-year-old cleaner presented to the GP complaining of a 2- week history of left arm pain that starts at the shoulder and runs down the arm to the fingertips. She reports intermittent tingling sensation in the left hand and has noticed some clumsiness such as dropping cups in the kitchen. Prior to this pain she had never experienced problems with her left arm. She is left-handed. Additionally, she suffers from severe headaches, which are often associated with neck pain. She has a background of asthma, which is well controlled.

What would you look for on examination (clinical findings from this case in bold):

Look/ inspection Cervical posture Previous scars Feel Tenderness of cervical musculature Crepitus Move Active Limited active range of motion of cervical spine Passive Assessment of passive range of motion of cervical spine is not expected for spinal examination Resisted Assessment of resisted movements of cervical spine is not expected for spinal examination but students will need to do a full neurological examination of the upper limbs where power will be checked Special tests for this Lhermitte sign: sensation of joint examination electrical on neck flexion Cervical compression test: laterally flex the head and apply downward pressure on it. A positive sign will be ipsilateral pain of the neck or shoulder. Other Left : Normal.

76

CERVICAL RADICULOPATHY Upper limb neurological examination: Reduced power left History: finger extension, reduced 62-year-old cleaner presented to the GP complaining of a 2- sensation left middle finger, left week history of left arm pain that starts at the shoulder and triceps jerk is absent runs down the arm to the fingertips. She reports intermittent tingling sensation in the left hand and has noticed some What are your differential diagnoses? clumsiness such as dropping cups in the kitchen. Prior to this • Cervical radiculopathy pain she had never experienced problems with her left arm. • Cervical spondylosis She is left-handed. Additionally, she suffers from severe • headaches, which are often associated with neck pain. She • Rotator cuff injury has a background of asthma, which is well controlled. Why is a cervical radiculopathy likely? What would you look for on examination (clinical findings • Typical history of pain shooting down the arm. from this case in bold): Patients may think the problem is in their “shoulder”. In this case the pain arises above the shoulder and Look/ inspection Cervical posture goes down to the fingers. Also on examination there Previous scars are positive neurological findings with an absent left Feel Tenderness of cervical triceps jerk (C7, C8) and weak finger extension (C7 musculature myotome) and reduced sensation to left middle finger Crepitus (C7 dermatome). There is also reduced ROM at the Move Active Limited active range of motion of cervical spine. All this suggests a radiculopathy of the cervical spine left C7 nerve root. This could be caused are a Passive Assessment of passive range of herniated cervical vertebral disc or osteophyte motion of cervical spine is not formation from facet joint degenerative changes which expected for spinal examination are impinging the nerve root Resisted Assessment of resisted movements of cervical spine is not expected for Why other diagnoses are unlikely? spinal examination but students will • Cervical spondylosis: It is difficult to distinguish need to do a full neurological between cervical spondylosis and cervical examination of the upper limbs radiculopathy from the history and examination. where power will be checked Cervical spondylosis is a degenerative disorder of the Special tests for this Lhermitte sign: sensation of cervical vertebral structures, which can cause a joint examination electrical shock on neck flexion spectrum of symptoms, from none, to neck pain, to Cervical compression test: laterally multi-level cervical radicular symptoms. The flex the head and apply downward degenerative disease can become very severe and pressure on it. A positive sign will be encroach upon the cervical spine, which can cause ipsilateral pain of the neck or more florid neurological symptoms, even of the lower shoulder. limbs. Therefore a radiculopathy can suggest Other Left shoulder examination: Normal.

76 77

spondylosis. Subsequent imaging with MRI will help distinguish the two conditions • Brachial plexus injury unlikely: No history of trauma. No signs of muscle wasting, no deformity left upper limb • Rotator cuff pathology: Would expect pain on resisted movements of glenohumeral joint

What investigations would you perform? • X-ray AP, lateral and oblique views of cervical spine (with flexion and extension views if there is suspicion for instability) • Degenerative changes of facet joints • Osteophyte formation • Disc space narrowing and endplate sclerosis • Assess alignment and spinal canal diameter • Look for foraminal stenosis caused by osteophytes. • MRI cervical spine: • T2 axial imaging is the modality of choice and gives needed information on the status of the soft tissues. You can expect to see disc degeneration and herniation, foraminal stenosis with nerve root compression (loss of perineural fat), central compression with CSF effacement • Refer to orthopaedics if confirmed diagnosis and neurological compromise • Will help distinguish from cervical spondylosis. • CT: Can give more information on bony anatomy e.g. osteophyte formation compressing neural elements. It is also used as a preoperative planning tool • CT myelography (has been largely replaced by MRI)

What are the management options? a) Initial management by the GP includes: • Analgesia as per WHO pain ladder (start with Paracetamol and NSAIDs) • Orthopaedic/ Neurosurgical referral as neurological findings

78

spondylosis. Subsequent imaging with MRI will help b) Orthopaedic team in secondary care: distinguish the two conditions i) Non-operative: • Brachial plexus injury unlikely: No history of trauma. • Rest, Analgesics, refer for physiotherapy, Spinal No signs of muscle wasting, no deformity left upper steroid/ anaesthetic injections limb ii) Operative: • Rotator cuff pathology: Would expect pain on resisted • Anterior cervical decompression and movements of glenohumeral joint • Posterior ; Cervical total disc replacement What investigations would you perform? • X-ray AP, lateral and oblique views of cervical spine (with flexion and extension views if there is suspicion for instability) • Degenerative changes of facet joints • Osteophyte formation • Disc space narrowing and endplate sclerosis • Assess alignment and spinal canal diameter • Look for foraminal stenosis caused by osteophytes. • MRI cervical spine: • T2 axial imaging is the modality of choice and gives needed information on the status of the soft tissues. You can expect to see disc degeneration and herniation, foraminal stenosis with nerve root compression (loss of perineural fat), central compression with CSF effacement • Refer to orthopaedics if confirmed diagnosis and neurological compromise • Will help distinguish from cervical spondylosis. • CT: Can give more information on bony anatomy e.g. osteophyte formation compressing neural elements. It is also used as a preoperative planning tool • CT myelography (has been largely replaced by MRI)

What are the management options? a) Initial management by the GP includes: • Analgesia as per WHO pain ladder (start with Paracetamol and NSAIDs) • Orthopaedic/ Neurosurgical referral as neurological findings

78 79

WHIPLASH INJURY

History: A 31-year-old lady presents to the GP two days following a road traffic accident (RTA). Her car was rear-ended. She was travelling at 120km/ hour, wearing her seatbelt, and on impact the airbags were deployed. She sustained no other injuries, and was completely asymptomatic following the accident. She attended the Emergency Department, had a normal cervical spine X-ray and was discharged. However she woke with severe neck pain and stiffness the following day and it has not eased since. She works as a teacher, has no background history and is on the combined oral contraceptive pill (COCP).

What would you look for on examination (clinical findings from this case in bold):

Look/ inspection Cervical posture – holding head stiffly Feel Tenderness of cervical musculature Check for tenderness over vertebrae (non-tender in this case) Move Active Limited active range of motion of cervical spine (Pain and reduced ROM secondary to same is usually the only positive sign found on exam) Passive Assessment of passive range of motion of cervical spine is not expected for spinal examination Resisted Assessment of resisted movements of cervical spine is not expected for spinal examination but students will need to do a full neurological examination of the upper limbs where power will be checked Special tests for this Nil joint examination Other Left shoulder examination: Normal.

80

WHIPLASH INJURY Upper limb neurological examination: normal History: A 31-year-old lady presents to the GP two days following a What are your differential diagnoses? road traffic accident (RTA). Her car was rear-ended. She was • Whiplash injury travelling at 120km/ hour, wearing her seatbelt, and on impact • Spinal fracture the airbags were deployed. She sustained no other injuries, • Cervical disc protrusion and was completely asymptomatic following the accident. She attended the Emergency Department, had a normal cervical Why is a whiplash injury likely? spine X-ray and was discharged. However she woke with • Typical history, post RTA, with associated pain limited severe neck pain and stiffness the following day and it has not active movement of neck and a normal cervical spine eased since. She works as a teacher, has no background X-ray history and is on the combined oral contraceptive pill (COCP). Why other diagnoses are unlikely? What would you look for on examination (clinical findings • Spinal fracture unlikely: Not likely due to delay in from this case in bold): onset of symptoms, non-tender vertebrae on examination and normal cervical spine X-ray in Look/ inspection Cervical posture – holding head emergency department. stiffly • Cervical disc protrusion unlikely: Normal neurological Feel Tenderness of cervical examination upper limbs musculature Check for tenderness over vertebrae What investigations would you perform? (non-tender in this case) • No more investigations indicated as no concerning Move Active Limited active range of motion of signs on clinical examination and had normal cervical cervical spine spine X-ray after the accident (Pain and reduced ROM secondary • If any diagnostic doubt, consider repeating X-ray to same is usually the only positive Cervical spine or arranging MRI cervical spine (or CT sign found on exam) if MRI contra-indicated). Check for disc Passive Assessment of passive range of bulge/herniation motion of cervical spine is not expected for spinal examination What are the management options? Resisted Assessment of resisted movements a) Initial management by the GP includes: of cervical spine is not expected for • Analgesia as per WHO pain ladder (start with spinal examination but students will Paracetamol and NSAIDs) need to do a full neurological • Orthopaedic referral not indicated (the majority of examination of the upper limbs patients' pain resolves in a few days. 50% of patients where power will be checked will continue to suffer from symptoms thereafter. Special tests for this Nil Treatment at this stage may require input from joint examination physiotherapists and or specialists). Other Left shoulder examination: Normal.

80 81

LUMBAR RADICULOPATHY

History: A 43-year-old male presents to his GP complaining of left buttock pain of 2 days duration which radiates down the back of his left leg below the knee. He describes recently moving house and feeling a ‘pop’ when lifting furniture followed by an acute onset of pain. On questioning he tells you the pain is burning in nature and constant. He does not have saddle anaesthesia, urinary retention or faecal incontinence. He does not feel weak in his legs, but pain is limiting his movements. He is otherwise fit and healthy.

What would you look for on examination (clinical findings from this case in bold):

Look/ inspection Antalgic gait Loss of lumbar lordosis Look for scoliosis/spinal scars Feel Tenderness over L5 centrally and left L5/S1 facet joint Move Active Pain-limited active range of motion of Lumbar Spine movements (non- specific) Passive Assessment of passive range of motion of lumbar spine is not expected for spinal examination Resisted Assessment of resisted movements of lumbar spine is not expected for spinal examination but students will need to do a full neurological examination of the lower limbs where power will be checked

82

LUMBAR RADICULOPATHY Special tests for Straight leg raise (SLR) testing – this joint limited on left side to 30 degrees History: examination (right side normal 90 degrees) A 43-year-old male presents to his GP complaining of left Other Hip examination: normal buttock pain of 2 days duration which radiates down the back Full lower limb neurological of his left leg below the knee. He describes recently moving examination: normal house and feeling a ‘pop’ when lifting furniture followed by an Digital was not acute onset of pain. On questioning he tells you the pain is indicated as no history of symptoms or burning in nature and constant. He does not have saddle signs of cauda equina* (faecal anaesthesia, urinary retention or faecal incontinence. He does incontinence, urinary retention or saddle not feel weak in his legs, but pain is limiting his movements. anaesthesia) He is otherwise fit and healthy. Full neurological examination lower limbs: normal What would you look for on examination (clinical findings from this case in bold): What are your differential diagnoses? • Lumbar radiculopathy Look/ inspection Antalgic gait • Hip osteoarthritis (from history) Loss of lumbar lordosis • Cauda Equina syndrome Look for scoliosis/spinal scars Feel Tenderness over L5 centrally and left Why is a lumbar radiculopathy likely? L5/S1 facet joint • Typical history with pain in back, going to buttock and Move Active Pain-limited active range of motion down back of left leg. Positive SLR of Lumbar Spine movements (non- specific) Why other diagnoses are unlikely? Passive Assessment of passive range of motion • Hip OA: Buttock pain of acute onset radiating down of lumbar spine is not expected for the left leg is more indicative of a radiculopathy. Hip spinal examination OA tends to present with groin pain of insidious onset Resisted Assessment of resisted movements of radiating down anterior thigh to knee lumbar spine is not expected for spinal • Cauda Equina* Syndrome: Cauda Equina usually examination but students will need to do causes bilateral leg pain. Denies saddle anaesthesia, a full neurological examination of the urinary retention or faecal incontinence lower limbs where power will be checked What investigations would you do? The GP may not perform any investigations if high diagnostic likelihood of acute lumbar radiculopathy e.g. from lumbar disc disease (). If there are neurological signs or if there is a diagnostic doubt, the following investigations can be considered: • X-ray: AP lumbar Spine • Loss of lordosis

82 83

• Loss of disc height • Lumbar spondylosis • MRI Spine • (Specific indications for MRI include pain lasting > one month and not responding to non-operative management, infection (IV drug user, h/o of fever and chills), concerning history regarding malignancy, cauda equina syndrome (bowel/bladder changes)) • MRI imaging will help identify disc herniation, nerve root foraminal impingement, spinal canal stenosis.

What are the management options? a) Initial management by the GP includes conservative treatments: • Education regarding natural history of sciatica (often takes months to resolve). • Rest • Analgesia as per WHO ladder • Refer for physiotherapy • Referral for orthopaedic or neurosurgical input if a) severe and incapacitating pain and/or b) significant neurological findings on peripheral examination lower limbs (e.g. motor weakness, numbness, absent ) • Immediate referral to emergency department for orthopaedic or neurosurgical input if symptoms or signs of cauda equina* (surgery needed within 48 hours). b) Orthopaedic/Neurosurgical management in secondary care: • Non-operative: Analgesia, Physiotherapy, Corticosteroid Injections (Epidural, selective nerve root block) • Operative: • • Micro-discectomy

84

• Loss of disc height SPONDYLOLISTHESIS • Lumbar spondylosis • MRI Spine History: • (Specific indications for MRI include pain 45-year-old housewife presents to GP on a Monday morning lasting > one month and not responding to with worsening back pain over previous 2 years despite non-operative management, infection (IV drug having taken analgesics and trying physiotherapy. Over the user, h/o of fever and chills), concerning weekend pain has become more severe with shooting pain history regarding malignancy, cauda equina down to both buttocks, causing her significant distress. She syndrome (bowel/bladder changes)) denies any history of recent trauma or bowel or bladder • MRI imaging will help identify disc herniation, disturbance. She reports having had an X-ray 2 years ago and nerve root foraminal impingement, spinal remembers “something unusual” on the report. You look up canal stenosis. the X-ray report and note an incidental finding of a pars defect of L5 on X-ray of lumbar spine. What are the management options? a) Initial management by the GP includes conservative What would you look for on examination (clinical findings treatments: from this case in bold): • Education regarding natural history of sciatica (often takes months to resolve). Look/ inspection Possible waddling gait • Rest Hyper-lordotic lumbar spine • Analgesia as per WHO ladder Gluteal • Refer for physiotherapy Feel Nil of note • Referral for orthopaedic or neurosurgical input if a) Move Active Decreased forward flexion (Schober severe and incapacitating pain and/or b) significant test*) neurological findings on peripheral nervous system Passive Assessment of passive range of examination lower limbs (e.g. motor weakness, motion of lumbar spine is not expected numbness, absent reflexes) for spinal examination • Immediate referral to emergency department for Resisted Assessment of resisted movements of orthopaedic or neurosurgical input if symptoms or lumbar spine is not expected for spinal signs of cauda equina* (surgery needed within 48 examination but students will need to hours). do a full neurological examination of b) Orthopaedic/Neurosurgical management in secondary the lower limbs where power will be care: checked • Non-operative: Analgesia, Physiotherapy, Special tests for this Straight leg raise (restricted Corticosteroid Injections (Epidural, selective nerve joint examination bilaterally) root block) Trendelenberg test*: positive • Operative: Other Full neurological examination lower • Laminectomy limbs: normal • Discectomy What are your differential diagnoses? • Micro-discectomy • Spondylolisthesis

84 85

• Spondyloarthropathy • Disc prolapse/ Herniated Nucleus Pulposis (HNP) • Malignancy

Why is spondylolisthesis likely? • Symptoms tend to occur bilaterally, hyperlordosis often associated with this condition, as is reduced forward flexion

Why other diagnoses are unlikely? • Disc prolapse: Symptoms tend to be unilateral • Spondyloarthropathy: Broad term for disease of the spine, including ankylosing spondylitis and Reiter's syndrome. These tend to present with low back/buttock pain, with morning stiffness that is relieved somewhat after , and generalised fatigue • Malignancy Questions in the history regarding unintentional weight loss, smoking history, chronic , bloody sputum, history of cancer. Beware Multiple Myeloma as a cause of back pain – if concern due to rest pain, Bence Jones protein and SPEP (serum protein electrophoresis)

What investigations would you do? • X-ray: AP and lateral, oblique and flexion extension views of lumbar spine • Frank spondylolisthesis on lateral • Pars defect on oblique • Check Pelvic incidence (sacral slope + pelvic tilt) – correlates with severity of disease • Instability on flexion extension (4mm translation compared to adjacent segment) • MRI Lumbosacral spine: • Check for neural compression/ foraminal stenosis.

What are the management options? a) Initial management by the GP includes conservative options:

86

• Spondyloarthropathy • Analgesia as per WHO pain ladder (start with • Disc prolapse/ Herniated Nucleus Pulposis (HNP) Paracetamol and NSAIDs) • Malignancy • Refer for physiotherapy: may include hamstring stretching, core strength, lumbar flexion exercises Why is spondylolisthesis likely? • Referral to Orthopaedic secondary care if symptoms • Symptoms tend to occur bilaterally, hyperlordosis fail to resolve often associated with this condition, as is reduced b) Orthopaedic secondary care management: forward flexion • Non-operative: Analgesia/ Physiotherapy/ Bracing in the acute phase Why other diagnoses are unlikely? • Operative: Foraminal decompression; In situ fusion • Disc prolapse: Symptoms tend to be unilateral • Spondyloarthropathy: Broad term for disease of the spine, including ankylosing spondylitis and Reiter's syndrome. These tend to present with low back/buttock pain, with morning stiffness that is relieved somewhat after exercise, and generalised fatigue • Malignancy Questions in the history regarding unintentional weight loss, smoking history, chronic cough, bloody sputum, history of cancer. Beware Multiple Myeloma as a cause of back pain – if concern due to rest pain, Bence Jones protein and SPEP (serum protein electrophoresis)

What investigations would you do? • X-ray: AP and lateral, oblique and flexion extension views of lumbar spine • Frank spondylolisthesis on lateral • Pars defect on oblique • Check Pelvic incidence (sacral slope + pelvic tilt) – correlates with severity of disease • Instability on flexion extension (4mm translation compared to adjacent segment) • MRI Lumbosacral spine: • Check for neural compression/ foraminal stenosis.

What are the management options? a) Initial management by the GP includes conservative options:

86 87

LUMBAR PAIN (WITH POSSIBLE RED FLAGS)

History: 72-year-old retired administrator with a 60-pack year smoking history presents with sudden onset non-severe (3-4/10) thoracolumbar pain one week previously. She has a positive family history of osteoporosis (mother) and a history of osteopenia on DEXA 5 years ago. She has been taking calcium and Vitamin D since then but did not have a repeat DEXA as advised two years ago.

You ask about other ‘red flags’ for back pain. She has no history of previous cancer. She denies recent trauma or falls. She does get some pain at night since this started 1 week ago.

What would you look for on examination (clinical findings from this case in bold):

Look/ inspection Thoracic kyphosis. BMI – low (18) in this case. Feel Tenderness over thoracolumbar spine centrally Move Active Painful range of motion of Thoracic and Lumbar spine Passive Assessment of passive range of motion of Thoracic and Lumbar spine is not expected for spinal examination Resisted Assessment of resisted movements of Thoracic and Lumbar spine is not expected for spinal examination but students will need to do a full neurological examination of the lower limbs where power will be checked Special tests for this Nil joint examination Other Full lower limb neurological examination: Normal Hip examination bilaterally: Normal Digital rectal examination was not

88

LUMBAR PAIN (WITH POSSIBLE RED FLAGS) performed as no history of other symptoms or signs of cauda equina* History: (if a male patient, could consider DRE 72-year-old retired administrator with a 60-pack year smoking if prostate cancer was suspected). history presents with sudden onset non-severe (3-4/10) thoracolumbar pain one week previously. She has a positive What are your differential diagnoses? family history of osteoporosis (mother) and a history of • Vertebral compression fracture osteopenia on DEXA 5 years ago. She has been taking • Primary or secondary tumour of the spine calcium and Vitamin D since then but did not have a repeat • Facet joint degeneration DEXA as advised two years ago. • Non-specific low back pain

You ask about other ‘red flags’ for back pain. She has no Why is a vertebral compression fracture (caused by history of previous cancer. She denies recent trauma or falls. osteoporosis) likely? She does get some pain at night since this started 1 week • This is most likely given the history of osteopenia, low ago. BMI, thoracic kyphosis, tenderness and pain on active ROM thoracolumbar spine. However further What would you look for on examination (clinical findings investigations will be needed to exclude the from this case in bold): differential diagnoses

Look/ inspection Thoracic kyphosis. Why other diagnoses are unlikely? BMI – low (18) in this case. • Primary or secondary tumour of the spine: It is very Feel Tenderness over thoracolumbar important to rule these out (note smoking history), spine centrally with bloods (ESR, CRP, Bone profile, Serum protein Move Active Painful range of motion of Thoracic electrophoresis, Bence Jones proteins) and X-rays of and Lumbar spine thoracic and lumbar spine Passive Assessment of passive range of • Facet joint degeneration is unlikely: Arthritic changes motion of Thoracic and Lumbar spine of the facet joints are also a possibility. Pain onset is is not expected for spinal examination usually gradual though Resisted Assessment of resisted movements of • Non-specific low back pain unlikely: This is the most Thoracic and Lumbar spine is not common underlying symptom for back pain. Other expected for spinal examination but causes should be ruled out. This history points to an students will need to do a full osteoporotic fracture neurological examination of the lower limbs where power will be checked What investigations would you do? Special tests for this Nil • Bloods: Full Blood Count, Renal Profile, ESR, CRP, joint examination Liver Profile, Bone Profile, Serum Protein Other Full lower limb neurological Electrophoresis (consider multiple myeloma), PSA (in examination: Normal males), Vitamin D, PTH Hip examination bilaterally: Normal Digital rectal examination was not

88 89

• X-ray thoracic and lumbar spine and SI joint: (check for osteoporotic fracture (collapse of vertebral body) and/or wedge fracture, vertebral metastases) • X-ray Chest (if neoplasia is a possibility) • DEXA scan (if osteoporosis likely) • MRI Spine: if concerned about other causes or red flags e.g. if there is a neurological deficit an MRI would be required for assessment of any impact that retropulsion may have on the spinal cord (spinal cord compression above L2, or cauda equina at or below this level)

What are the management options? a) Initial management by the GP includes conservative options: • Analgesia as per WHO pain ladder (start with Paracetamol and NSAIDs) • Await results of investigations before referring for Physiotherapy (may not be indicated) • Osteoporosis management: Oral Calcium and Vitamin D supplementation. Bisphosphonates recommended for osteoporosis • Referral to Orthopaedic secondary care if indicated (e.g. osteoporotic fracture confirmed) b) Orthopaedic secondary care management: i) Non-operative: • Analgesia • Refer for physiotherapy: Bracing can be used to provide support while the fracture heals. Hyperextension braces can be used to prevent progression, but are uncomfortable • Osteoporosis management: Oral Calcium and Vitamin D supplementation. Bisphosphonates recommended for osteoporosis. Denosumab injections every 6 months for women who are intolerant of bisphosphonates. PTH-analogue injections (Forsteo) recommended for 18 months if vertebral compression fracture ii) Operative options for thoracolumbar burst fracture:

90

• X-ray thoracic and lumbar spine and SI joint: (check • Kyphoplasty* is indicated if pain persists for > 6 for osteoporotic fracture (collapse of vertebral body) weeks with non-operative treatment and/or wedge fracture, vertebral metastases) • Spinal Stabilisation may be required if the posterior • X-ray Chest (if neoplasia is a possibility) longitudinal ligament (PLL) is compromised, or if the • DEXA scan (if osteoporosis likely) fracture is unstable • MRI Spine: if concerned about other causes or red flags e.g. if there is a neurological deficit an MRI would be required for assessment of any impact that retropulsion may have on the spinal cord (spinal cord compression above L2, or cauda equina at or below this level)

What are the management options? a) Initial management by the GP includes conservative options: • Analgesia as per WHO pain ladder (start with Paracetamol and NSAIDs) • Await results of investigations before referring for Physiotherapy (may not be indicated) • Osteoporosis management: Oral Calcium and Vitamin D supplementation. Bisphosphonates recommended for osteoporosis • Referral to Orthopaedic secondary care if indicated (e.g. osteoporotic fracture confirmed) b) Orthopaedic secondary care management: i) Non-operative: • Analgesia • Refer for physiotherapy: Bracing can be used to provide support while the fracture heals. Hyperextension braces can be used to prevent progression, but are uncomfortable • Osteoporosis management: Oral Calcium and Vitamin D supplementation. Bisphosphonates recommended for osteoporosis. Denosumab injections every 6 months for women who are intolerant of bisphosphonates. PTH-analogue injections (Forsteo) recommended for 18 months if vertebral compression fracture ii) Operative options for thoracolumbar burst fracture:

90 91

NON-SPECIFIC LOW BACK PAIN

History: A 24-year-old accountant presents to his GP clinic with a six- month history of pain in the lumbar spine region. This has been getting progressively worse over the past few weeks and he has had to stop playing rugby because of it. He complains of a very stiff back when he wakes up in the morning. He took one of his mother’s diazepam tablets and found it gave him great relief. He is otherwise healthy. There is no history of trauma to his spine (other than playing rugby). He is otherwise well and has no medical history of note.

What would you look for on examination (clinical findings from this case in bold):

Look/ inspection Loss of lumbar lordosis, Assess for scoliosis/spinal scars Feel Tenderness over lumbar area. Feel along spinous processes and paraspinal muscles. Move Active Pain limited active range of motion of Lumbar Spine movements. Schober test to establish loss of flexion of lumbar spine FABER test to check sacro-iliac joints Passive Assessment of passive range of motion of lumbar spine is not expected for spinal examination* Resisted Assessment of resisted movements of lumbar spine is not expected for spinal examination* but students will need to do a full neurological examination of the lower limbs where power will be checked Special tests for this Straight leg raise (SLR) testing – joint examination normal Wall Test* Other Full lower limb neurological examination – normal

92

NON-SPECIFIC LOW BACK PAIN What are your differential diagnoses? History: • Non-specific low back pain A 24-year-old accountant presents to his GP clinic with a six- • Herniated nucleus pulposus/ disc prolapse month history of pain in the lumbar spine region. This has • Spondylolisthesis been getting progressively worse over the past few weeks and • Spondyloarthopathy he has had to stop playing rugby because of it. He complains of a very stiff back when he wakes up in the morning. He took Why is a mechanical lumbosacral pain likely? one of his mother’s diazepam tablets and found it gave him • This is likely given the young age of the patient who is great relief. He is otherwise healthy. There is no history of involved in regular physical activity. Further evidence trauma to his spine (other than playing rugby). He is otherwise is from the fact that pain is relieved with a muscle well and has no medical history of note. relaxant

What would you look for on examination (clinical findings Why other diagnoses are unlikely? from this case in bold): • Prolapsed disc unlikely: As there is no and no neurological signs Look/ inspection Loss of lumbar lordosis, • Spondylolisthesis unlikely: As there is no bilateral Assess for scoliosis/spinal scars radicular pain Feel Tenderness over lumbar area. • Spondyloarthropathy unlikely: Broad term for disease Feel along spinous processes and of the spine, including ankylosing spondylitis and paraspinal muscles. Reiter's syndrome. These tend to present with low Move Active Pain limited active range of motion back/buttock pain, with morning stiffness that is of Lumbar Spine movements. relieved somewhat after exercise, and with Schober test to establish loss of generalised fatigue. This should be considered in any flexion of lumbar spine young man presenting with lumbar discomfort FABER test to check sacro-iliac joints Passive Assessment of passive range of What investigations would you do? motion of lumbar spine is not expected • If mechanical low back pain is suspected, there is for spinal examination* usually no indication for blood or radiological Resisted Assessment of resisted movements of investigations lumbar spine is not expected for spinal • If ankylosing spondylitis or other spondyloarthropathy examination* but students will need to considered, plain film of the spine and SI joints do a full neurological examination of warranted the lower limbs where power will be • Of note, MRI has a high rate of incidental findings. checked Symptoms are not consistent with any pathology that Special tests for this Straight leg raise (SLR) testing – would require an operative intervention therefore any joint examination normal MRI finding would be academic and potentially Wall Test* unnecessarily worrisome Other Full lower limb neurological examination – normal What are the management options?

92 93

a) Initial management by the GP includes conservative options: • Analgesia as per WHO pain ladder (start with Paracetamol and NSAIDs) • Refer for physiotherapy: core stability and flexibility (encourage yoga, pilates etc. • Encourage not taking medications that have been prescribed for somebody else • If symptoms persist or any change in symptoms get patient to come back (and can consider bloods and/ or radiological assessment then). b) Orthopaedic referral is not indicated in this case.

94 a) Initial management by the GP includes conservative SPINAL STENOSIS options: • Analgesia as per WHO pain ladder (start with History: Paracetamol and NSAIDs) A 73-year-old man, who used to work as a brick-layer, • Refer for physiotherapy: core stability and flexibility presents to his GP with a 6 month history of bilateral buttock (encourage yoga, pilates etc. and leg pain that is worse with prolonged standing and • Encourage not taking medications that have been relieved with sitting. He finds relief when he goes shopping prescribed for somebody else and is pushing a trolley. He is surprised that he is able to • If symptoms persist or any change in symptoms get exercise comfortably on his stationary bike at home. He patient to come back (and can consider bloods and/ denies neurological symptoms of his lower limbs otherwise. or radiological assessment then). He denies urinary retention, saddle anaesthesia or faecal b) Orthopaedic referral is not indicated in this case. incontinence.

What would you look for on examination (clinical findings from this case in bold):

Look/ inspection Loss of lumbar lordosis, no obvious scoliosis/spinal scars Sagittal and coronal balance Feel Possible tenderness over lumbar area Move Active Possible limited active range of motion of Lumbar Spine movements (non-specific) Passive Assessment of passive range of motion of lumbar spine is not expected for spinal examination Resisted Assessment of resisted movements of lumbar spine is not expected for spinal examination but students will need to do a full neurological examination of the lower limbs where power will be checked Special tests for this Straight leg raise (SLR) testing – joint examination possible limitation bilaterally, but not typically positive Wall Test- may establish loss lumbar lordosis Other Full lower limb neurological examination – normal in this case

94 95

Peripheral vascular examination lower limbs - normal

What are your differential diagnoses? • Spinal stenosis/ Neurogenic claudication. Spinal stenosis is the narrowing of the spinal canal or neural foramina, which produces nerve root compression, nerve root ischaemia, and variable symptoms from back pain to radicular pain • Vascular claudication • Spondylolisthesis • Prolapsed disc

Why is spinal stenosis likely? • Pain on standing/walking, relieved by sitting. Loss of lumbar lordosis

Why other diagnoses are unlikely? • Vascular claudication unlikely*: Can be difficult to distinguish from neurogenic claudication. Normal peripheral vascular examination. • Spondylolisthesis unlikely: As you would expect a much longer pain history and the patient to have an increase in lumbar lordosis. • Prolapsed disc unlikely: As normal lower limb neurological examination.

What investigations would you do? Refer for X-ray and/ or MRI: • X-ray lumbosacral spine: Standing AP and lateral. We are looking for nonspecific degenerative findings (disk space narrowing, osteophyte formation), degenerative scoliosis, spondylolisthesis or spondylolysis. This may be clearer with flexion extension views, where you would be looking for an increased translation of one segment over another • MRI: Check for central stenosis, obliteration of the thecal sac or perineural fat, degenerative changes of the facet joints leading to osteophyte formation, ligamentum flavum hypertrophy

96

Peripheral vascular examination lower • CT: If MRI contraindicated a CT myelogram may be limbs - normal performed. Disruption in radio opaque dye may be seen on imaging What are your differential diagnoses? • Spinal stenosis/ Neurogenic claudication. Spinal What are the management options? stenosis is the narrowing of the spinal canal or neural a) Initial management by the GP includes conservative foramina, which produces nerve root compression, options: nerve root ischaemia, and variable symptoms from • Analgesia as per WHO pain ladder (start with back pain to radicular pain Paracetamol and NSAIDs) • Vascular claudication • Refer for physiotherapy • Spondylolisthesis • Orthopaedic referral if symptoms difficult to manage • Prolapsed disc b) Orthopaedic team in secondary care: i) Non-operative: Why is spinal stenosis likely? • Rest, Analgesics, Physiotherapy, Spinal steroid • Pain on standing/walking, relieved by sitting. Loss of injections lumbar lordosis ii) Operative: • Decompression with or without fusion, indicated with Why other diagnoses are unlikely? failed conservative measures, or if there is a • Vascular claudication unlikely*: Can be difficult to progressive neurological deficit. distinguish from neurogenic claudication. Normal • Fusion required if there is segmental instability (pre- peripheral vascular examination. existing or iatrogenic) • Spondylolisthesis unlikely: As you would expect a much longer pain history and the patient to have an increase in lumbar lordosis. • Prolapsed disc unlikely: As normal lower limb neurological examination.

What investigations would you do? Refer for X-ray and/ or MRI: • X-ray lumbosacral spine: Standing AP and lateral. We are looking for nonspecific degenerative findings (disk space narrowing, osteophyte formation), degenerative scoliosis, spondylolisthesis or spondylolysis. This may be clearer with flexion extension views, where you would be looking for an increased translation of one segment over another • MRI: Check for central stenosis, obliteration of the thecal sac or perineural fat, degenerative changes of the facet joints leading to osteophyte formation, ligamentum flavum hypertrophy

96 97

APPENDICES

98

APPENDIX 1: IMPORTANT FACTS AND FIGURES APPENDICES AROUND SPINAL HISTORY AND EXAMINATION

Cauda Equina Syndrome (CES) CES is a surgical emergency. It is not common, with an estimated incidence of 1: 65,000 per year. The cause of the compression of the cauda equina may be: 1. Disc herniation (most common) 2. Spinal stenosis 3. Tumours 4. Trauma (retropulsion of fracture fragment) 5. Spinal epidural haematoma 6. Epidural abscess

The words cauda equina derive from the latin term, “horse’s tail”; a collection of peripheral within the spinal canal. Any impingement or lesion of the cauda equina is thus considered a lower motor neuron lesion.

A syndrome is a constellation of symptoms. CES symptoms are: • Pain: Back of and legs • Numbness: Saddle area, back of legs and soles of feet • Weakness: motor weakness in lower limbs • Atrophy: of calves if insidious onset • Paralysis of dysfunction of bladder and bowel • Absence of lower limb reflexes • Decreased rectal tone

Important to note that onset may be insidious as well as acute.

Don’t forget: • Any person presenting with complaints of back pain should be questioned about a past history of cancer or symptoms consistent with an undiagnosed neoplasm.

98 99

What Cancers can metastasise to bone? • Breast • Lung • Prostate • Thyroid • Renal • Colon

Where are the most common sites for tumour metastasis? • Lung • Liver • Bone a. Axial skeleton (thoracic spine most commonly affected) b. Proximal limb girdle (proximal femur most commonly affected)

Prognosis If metastatic bone disease is detected, the median survival for the following cancers is: • Thyroid cancer: 48 months • Prostatic Adenocarcinoma: 40 months • Breast cancer: 24 months • Renal cell carinoma: Variable but can be 6 months • Lung cancer: 6 months

100

What Cancers can metastasise to bone? • Breast RED FLAGS1 YELLOW FLAGS • Lung • Prostate (HELP IDENTIFY (INDICATE • Thyroid POTENTIALLY SERIOUS PSYCHOSOCIAL • Renal CONDITIONS) BARRIERS TO • Colon RECOVERY)

Where are the most common sites for tumour Features of cauda equina Belief that pain and activity metastasis? syndrome are harmful • Lung Severe worsening pain, Low or negative moods. • Liver especially at night/when lying Social withdrawal • Bone down a. Axial skeleton (thoracic spine most commonly History of cancer Over reaction affected) Thoracic pain Non-organic distribution b. Proximal limb girdle (proximal femur most (non-dermatomal sensory commonly affected) loss) Structural deformity Inconsistent signs Prognosis History of violent trauma Superficial non-anatomical If metastatic bone disease is detected, the median survival for tenderness the following cancers is: Age < 20 or > 55 years Pain with simulated testing • Thyroid cancer: 48 months (e.g. axial loading or pelvic • Prostatic Adenocarcinoma: 40 months rotation) • Breast cancer: 24 months Neurological disturbance Problems with claim and • Renal cell carinoma: Variable but can be 6 months compensation • Lung cancer: 6 months Fever, night sweats, weight loss Problems at work, poor job satisfaction History of immunosuppression e.g. HIV, drug abuse, prolonged use of steroids

1 Accessed from http://www.sheffieldbackpain.com/professional-

resources/learning/in-detail/yellow -flags-in -back-pain (on 4th July 2016)

100 101

APPENDIX 2: DIAGNOSTIC CRITERIA IN MUSCULOSKELETAL MEDICINE

1. Neck of femur fractures:

Rarely need further imaging aside from initial X-ray. However, laboratory investigations pertaining to bone health need to be done when the patient is an inpatient, and input from an orthogeriatrician (where available) is invaluable. A follow up DEXA scan is recommended.

Garden classification: • Type 1: Incomplete/Valgus impacted • Type 2: Complete, non-displaced on AP and lateral views • Type 3: Complete with partial displacement (look for discontinuation of trabecular pattern) • Type 4: Complete displacement

2. Kocher criteria (for septic arthritis in paediatric population)

1) Fever >38.5 2) WCC >12000mm3 3) Non-weight-bearing on affected side 4) ESR > 40mm/h

• When all criteria are present met, there is a 99% probability of septic arthritis • If 3/4 criteria are present, there is a 93% probability of septic arthritis • With 2/4 criteria present, there is a 40% probability of septic arthritis • With 1/4 criteria being present, there is a 3% probability of septic arthritis

102

APPENDIX 2: DIAGNOSTIC CRITERIA IN 3. Steinburg classification for avascular necrosis MUSCULOSKELETAL MEDICINE Stage Pain X-ray image MRI Image

1. Neck of femur fractures: Stage I + Normal +

Rarely need further imaging aside from initial X-ray. However, Stage II + Changes on bone + laboratory investigations pertaining to bone health need to be trabeculate with sclerosis or osteolysis areas done when the patient is an inpatient, and input from an orthogeriatrician (where available) is invaluable. A follow up Stage III + Osteochondral fracture + DEXA scan is recommended. with sequestrum and flattening of the head Garden classification: Stage IV + Advanced lesions + • Type 1: Incomplete/Valgus impacted

• Type 2: Complete, non-displaced on AP and lateral views 4. Wells Criteria • Type 3: Complete with partial displacement (look for discontinuation of trabecular pattern) Wells criteria for scoring a DVT: • Type 4: Complete displacement • <2 DVT unlikely • >2 DVT Likely 2. Kocher criteria (for septic arthritis in paediatric population) Clinical Features Points

1) Fever >38.5 Active Cancer 1 2) WCC >12000mm3 Paralysis, paresis, recent immobilisation of lower 1 3) Non-weight-bearing on affected side extremities 4) ESR > 40mm/h Localised tenderness along deep venous system 1

• When all criteria are present met, there is a Entire leg swollen 1 99% probability of septic arthritis Calf swelling >3cm larger than asymptomatic side 1 • If 3/4 criteria are present, there is a 93% probability of septic arthritis unilateral pitting oedema 1 • With 2/4 criteria present, there is a 40% Collateral superficial 1 probability of septic arthritis Previously documented DVT 1 • With 1/4 criteria being present, there is a 3% probability of septic arthritis Alt diagnosis as likely or more than likely DVT 1

Alternate diagnosis as likely or more than likely -2 than DVT

102 103

5. Diagnostic criteria differentiating vascular from neurogenic claudication

Factors Neurogenic Vascular Evaluation after Increased Unchanged walking weakness Palliative factors Bending over, Stopping sitting Provocative Walking downhill Walking uphill factors Increased Increased lordosis metabolic demand Pulses Present Absent “Shopping cart” Present Absent sign Van gelderen No leg pain Leg pain bicycle test

104

5. Diagnostic criteria differentiating vascular from APPENDIX 3: ABBREVIATIONS neurogenic claudication

Factors Neurogenic Vascular ACL Anterior cruciate ligament Evaluation after Increased Unchanged ADHD Attention deficit hyperactivity disorder walking weakness ANCA Anti-neutrophil cyctoplasmic antibodies Palliative factors Bending over, Stopping CCP Cyclic citrullinated peptides sitting AROM Anterior range of motion Provocative Walking downhill Walking uphill AP Antero-posterior factors Increased Increased AVN Avascular necrosis lordosis metabolic BMI demand COCP Combined hormonal contraception Pulses Present Absent CRP C-reactive protein “Shopping cart” Present Absent CT Computed tomography sign DDH Developmental dysplasia of the hip Van gelderen No leg pain Leg pain DEXA Dual-energy X-ray absorptiometry bicycle test DRE Digital rectal examination DVT Deep vein thrombosis ECRB Extensor carpi radialis brevis ED Emergency Department ESR Erythrocyte sedimentation rate FBC Full blood count FFD Fixed flexion deformity

HNP Herniated nucleus pulposis INR International Normalised Ratio IV Intra-venous GP General Practitioner MUA Manipulation under anaesthesia MCL Medial collateral ligament MRI Magnetic resonance imaging NOACs Novel Oral Anticoagulants NSAIDs Non-steroidal anti-inflammatory drugs OA Osteoarthritis PCL Posterior collateral ligament PSA Prostate surface antigen PTH Parathyroid hormone PE PROM Posterior range of motion RA Rheumatoid arthritis

104 105

ROM Range of motion RTA Road traffic accident SLR Straight leg raise SPEP Serum protein electrophoresis SUFE Slipped Upper Femoral Epiphysis THR Total hip replacement WCC White cell count WHO World Health Organisation

106

ROM Range of motion APPENDIX 4: NOTES RTA Road traffic accident SLR Straight leg raise SPEP Serum protein electrophoresis SUFE Slipped Upper Femoral Epiphysis THR Total hip replacement WCC White cell count WHO World Health Organisation

106 107

108

108 109

110

110 111

112 113

112 113

114

114 115

116

116

RCSI Royal College of Surgeons in Ireland Coláiste Ríoga na Máinleá in Éirinn 123 St Stephen’s Green, Dublin 2 Tel: +353 1 402 2100 Email: [email protected] www.rcsi.com

EDUCATIONAL AND RESEARCH EXCELLENCE IN SURGERY MEDICINE PHARMACY PHYSIOTHERAPY NURSING & MIDWIFERY LEADERSHIP POSTGRADUATE STUDIES RADIOLOGY DENTISTRY SPORTS & EXERCISE MEDICINE